SlideShare a Scribd company logo
1 of 101
Download to read offline
Angina.com

            Angina.com
           March 24 2013
Research Interviews on Heart Disease
Coronary Artery Disease and Angina.
Angina.com
March 24 2013
Medical Disclaimer | Terms and Conditions


•   The contents of the Hemodialysis.com Site, such as text, graphics, images, and
    other material contained on the Hemodialysis.com Site ("Content") are for
    informational purposes only. The Content is not intended to be a substitute for
    professional medical advice, diagnosis, or treatment. Always seek the advice of
    your physician or other qualified health provider with any questions you may have
    regarding a medical condition. Never disregard professional medical advice or
    delay in seeking it because of something you have read on the Hemodialysis.com
    Site!
•   If you think you may have a medical emergency, call your doctor or 911
    immediately. Hemodialysis.com does not recommend or endorse any specific
    tests, physicians, products, procedures, opinions, or other information that may be
    mentioned on the Site. Reliance on any information provided by Hemodialysis.com
    or other Eminent Domains Inc (EDI) websites, EDI employees, others appearing on
    the Site at the invitation of Hemodialysis.com or EDI, or other visitors to the Site is
    solely at your own risk.
•   The Site may contain health- or medical-related materials that are sexually explicit.
    If you find these materials offensive, you may not want to use our Site. The Site
    and the Content are provided on an "as is" basis.



                              Read more interviews on Hemodialysis.com
Cardiac troponin I levels measured with a high-sensitive assay increase over time and are strong predictors of mortality in an elderly
                                                                 population.
                                            Angina.com Interview with Kai M Eggers, MD, PhD
                                      Department of Medical Sciences, Cardiology, Uppsala University
                                                              Uppsala, Sweden



•     Angina.com: What are the main findings of the study ?
•     Dr. Eggers: One main finding was that troponin I levels in our elderly study
      participants increased significantly over five years time. Troponin is reflective of
      myocardial abnormalities and in fact, the most pronounced increases were seen in
      subjects with evidence of heart disease or risk factors for its development.
      Accordingly, we found that subjects with higher troponin levels at baseline or
      greater increases had a significant risk for death, even when other risk factors or
      biomarkers were taken into account.
•     Another important result of our study was that we for the first time were able to
      measure troponin I levels in almost all of our study participants. This depends on
      the analytical performance of the applied assay from Abbott and is at difference to
      other studies in general populations in whom troponin remained undetectable in
      at least every third subject.
      Angina.com: Were any of the findings unexpected ?
•     Dr. Eggers: We suspected that troponin I would be a strong risk predictor but were
      surprised to see that it remained independently prognostic in analyses that were
      adjusted for CRP and NT-proBNP, biomarkers that have received much greater
      attention in studies performed in community populations.
Cardiac troponin I levels measured with a high-sensitive assay increase over time and are strong predictors of mortality in an elderly
                                                                 population.
                                            Angina.com Interview with Kai M Eggers, MD, PhD
                                      Department of Medical Sciences, Cardiology, Uppsala University
                                                              Uppsala, Sweden
                                                                    (cont)


•     Angina.com: Were any of the findings unexpected ?
•     Dr. Eggers: We suspected that troponin I would be a strong risk predictor but were surprised to see that it
      remained independently prognostic in analyses that were adjusted for CRP and NT-proBNP, biomarkers that have
      received much greater attention in studies performed in community populations.
•     Angina.com: What should clinicians and patients take home from this study ?
•     Dr. Eggers:Troponins are usually measured in patients with the suspicion of a heart attack. We have learned from
      our study that troponins can also be detected in presumably healthy subjects. This must be taken into account
      when interpreting troponin results in elderly patients who are admitted because of chest pain. Another important
      point is that troponin levels appear to increase with aging as result of age-related changes of the heart. This is
      discussed more in detail in another paper from our group published this month in Clinical Chemistry, and must
      probably be considered when defining reference levels for troponin.
•     Angina.com: What further research do you recommend as a result of your study?
•     Dr. Eggers: I would like to see a study investigating whether lifestyle changes or medical interventions could
      influence troponin levels, and whether this also might reflect better outcome. This issue has for example been
      investigated in an interesting paper from deFilippi and colleagues published last year in JACC demonstrating that
      physical exercise was related to a smaller increase in troponin levels over time. If we could show that troponin
      levels would mirror the beneficial cardiac effects of this kind of interventions, then this would be a big step
      towards personalized preventive strategies.
•     Cardiac troponin I levels measured with a high-sensitive assay increase over time and are strong predictors of
      mortality in an elderly population.
•     Citation:
•     Eggers KM, Venge P, Lindahl B, Lind L.
•     Department of Medical Sciences, Uppsala University, Sweden.
      J Am Coll Cardiol. 2013 Mar 6. pii: S0735-1097(13)00936-4. doi: 10.1016/j.jacc.2012.12.048. [Epub ahead of print]
Estimating the effect of long-term physical activity on cardiovascular disease and mortality: evidence from the Framingham Heart Study
                                                             Angina.com Interview with
                                                              Susan M. Shortreed PhD
                                     Assistant Investigator 
Biostatistics Unit, Group Health Research Institute
                                                 Group Health Research InstituteSeattle, WA 98101



•       Angina.com: What are the main findings of the study?
•       Dr. Shortreed: Physical activity throughout ones life is linked with longer life.

•       Angina.com: Were any of the findings unexpected?
•       Dr. Shortreed: The findings of this study confirmed that long-term physical activity has health
        benefits.
•       Our study followed individuals for a longer period of time than many previous studies in order to
        understand their physical activity habits and monitor their health outcomes.

•       Angina.com: What should clinicians and patients take away from this study?
•       Dr. Shortreed: Continued physical activity through out one’s life is important to a long
        life. Clinicians should talk with their patients about the benefits of physical activity and advise
        them on appropriate levels of physical activity considering their overall health. Patients should ask
        their clinician about the health benefits of physical activity and talk with their clinician before
        increasing their amount and intensity of physical activity.
•       The Centers for Disease Control and Prevention has some great online information and resources
        for both clinicians and patients on recommended physical activity levels for all ages.
Estimating the effect of long-term physical activity on cardiovascular disease and mortality: evidence from the Framingham Heart Study
                                                         Angina.com Interview with
                                                          Susan M. Shortreed PhD
                                 Assistant Investigator 
Biostatistics Unit, Group Health Research Institute
                                             Group Health Research InstituteSeattle, WA 98101
                                                                    (Cont)


• Angina.com: What further research do you recommend as a result
  of your study?
• Dr. Shortreed: We recommend studies investigate the long-term
  effects of physical activity in women and in diverse populations.
• To investigate the effects of long-term physical activity, we
  recommend studies record information on activity levels over time
  and use appropriate analytic methods to quantify the link between
  long-term effects of physical activity and health benefits.
• Citation:
• Estimating the effect of long-term physical activity on
  cardiovascular disease and mortality: evidence from the
  Framingham Heart Study
• Heart. 2013 Mar 8. [Epub ahead of print]
• Shortreed SM, Peeters A, Forbes AB.
  Monash University, , Melbourne, Australia.
Perioperative Dexmedetomidine Improves Outcomes of Cardiac Surgery
                              Angina.com Interview with Hong Liu, MD
             Anesthesiology and Pain Medicine, University of California Davis Health System,
                           4150 V Street Suite 1200, Sacramento, CA, 95817




• Angina.com: What are the main findings of the study?
• Dr. Liu: Perioperative dexmedetomidine use in cardiac surgical
  patients were more likely to have better in-hospital, 30-day and 1-
  year survival. The perioperative use of dexmedetomidine is
  also associated with a significant decrease in the incidence of
  postoperative overall complications and delirium.
• There was no evidence of adverse hemodynamic side effects of
  dexmedetomidine in patients undergoing cardiac surgery.

• Angina.com: Were any of the findings unexpected?
• Dr. Liu: Our study did not find perioperative dexmedetomidine
  improving postoperative kidney function.
• On the contrary, it worsened postoperative renal function.
Perioperative Dexmedetomidine Improves Outcomes of Cardiac Surgery
                                 Angina.com Interview with Hong Liu, MD
                Anesthesiology and Pain Medicine, University of California Davis Health System,
                              4150 V Street Suite 1200, Sacramento, CA, 95817
                                                    (cont)


•   Angina.com: What should clinicians and patients take away from this study?
•   Dr. Liu: Dexmedetomidine used in clinical practice have many desirable effects
    including analgesia, anxiolysis, inhibition of central sympathetic outflow and
    reduction of systemic norepinephrine release that improve hemodynamic stability,
    positively affect myocardial oxygen supply and demand and may provide
    myocardial protection. It also increased urine output. If indicated, physicians
    should consider use it.
•   Because of the above-mentioned properties of this medication, patients may have
    bradycardia and polyuria right after surgery.
•   Angina.com: What further research do you recommend as a result of your study?
•   Dr. Liu: Multicenter randomized, prospective trial. Dose-dependent on kidney
    protection
•   Citation:
•   Perioperative Dexmedetomidine Improves Outcomes of Cardiac Surgery
•   Fuhai Ji, Zhongmin Li, Hung Nguyen, Nilas Young, Pengcai Shi, Neal Fleming, and
    Hong Liu
•   Circulation. 2013;CIRCULATIONAHA.112.000936published online before print
    March 19 2013, doi:10.1161/CIRCULATIONAHA.112.000936
Effect of Intensive versus Moderate Lipid-Lowering Therapy on Epicardial Adipose Tissue in Hyperlipidemic Postmenopausal Women: A
                            Substudy of the BELLES (Beyond Endorsed Lipid Lowering with EBT Scanning) Trial)
                                 Angina.com Interview with Paolo Raggi, MD, FACC, FACP, FASNC, FCCT
                                              Director, Mazankowski Alberta Heart Institute
                                               Professor of Medicine, University of Alberta
                                   Capital Health Chair in Cardiac Sciences Edmonton, AB T6G 2B7, Ca



• Angina.com: What are the main findings of the study?
• Dr. Raggi: The present study was a sub-analysis of the BELLES study,
  originally designed to assess the effect of intensive versus moderate
  lipid-lowering therapy on coronary artery calcium in post-
  menopausal women.
• In the present sub-analysis, the effect on epicardial adipose tissue
  (EAT) was examined; EAT has recently emerged as a local source of
  inflammation and a surrogate and a possible active component in
  the pathogenesis of coronary atherosclerosis and of the formation
  of vulnerable atherosclerotic plaques. The main findings of the
  present sub-analysis are that although both treatment strategies
  resulted in halting the progression of EAT increase with age, only
  intensive lipid-lowering therapy was associated with a decrease in
  EAT volume.
Effect of Intensive versus Moderate Lipid-Lowering Therapy on Epicardial Adipose Tissue in Hyperlipidemic Postmenopausal Women: A
                            Substudy of the BELLES (Beyond Endorsed Lipid Lowering with EBT Scanning) Trial)
                                 Angina.com Interview with Paolo Raggi, MD, FACC, FACP, FASNC, FCCT
                                              Director, Mazankowski Alberta Heart Institute
                                               Professor of Medicine, University of Alberta
                                   Capital Health Chair in Cardiac Sciences Edmonton, AB T6G 2B7, Ca
                                                                  (cont)


• Angina.com: Were any of the findings unexpected?
• Dr. Raggi: The decrease in EAT volume was not associated with the
  decrease in LDL (bad) cholesterol. This mostly unexpected finding
  denotes that the beneficial effect of statin therapy on EAT is
  mediated through other mechanisms than LDL lowering; the most
  plausible mechanism is through the well-known anti-inflammatory
  effects of statins.
• Angina.com: What should clinicians and patients take away from
  this study?
• Dr. Raggi: This study provides another possible link between the
  beneficial effects of statins on cardiovascular risk; i.e. EAT volume
  modulation. It also provides possible evidence for a greater
  beneficial effect of intensive versus moderate lipid-lowering
  therapy. Finally, EAT may be used as surrogate of the effectiveness
  of risk-reduction therapies.
Effect of Intensive versus Moderate Lipid-Lowering Therapy on Epicardial Adipose Tissue in Hyperlipidemic Postmenopausal Women: A
                               Substudy of the BELLES (Beyond Endorsed Lipid Lowering with EBT Scanning) Trial)
                                    Angina.com Interview with Paolo Raggi, MD, FACC, FACP, FASNC, FCCT
                                                 Director, Mazankowski Alberta Heart Institute
                                                  Professor of Medicine, University of Alberta
                                      Capital Health Chair in Cardiac Sciences Edmonton, AB T6G 2B7, Ca



•      Angina.com: What further research do you recommend as a result of your study?
•      Dr. Raggi: A study of statin therapy with both EAT measurement pre- and post-
       treatment with adequate number of patients and follow-up to assess both
       surrogate (i.e. EAT volume change) and clinical events (major adverse
       cardiovascular events); if positive, this would establish the role of EAT volume
       measurement as a surrogate of clinical outcome.

•      Citation:
•      Effect of Intensive versus Moderate Lipid-Lowering Therapy on Epicardial Adipose
       Tissue in Hyperlipidemic Postmenopausal Women: A Substudy of the BELLES
       (Beyond Endorsed Lipid Lowering with EBT Scanning) Trial)
•      Alexopoulos N, Melek BH, Arepalli CD, Hartlage GR, Chen Z, Kim S, Stillman AE,
       Raggi P.
•      Division of Cardiology, Department of Medicine, Emory University, Atlanta, GA,
       USA.
       J Am Coll Cardiol. 2013 Mar 13. pii: S0735-1097(13)01002-4. doi:
       10.1016/j.jacc.2012.12.051. [Epub ahead of print]
Severity of coronary atherosclerosis in patients with a first acute coronary event: a diabetes paradox
                                  Angina.com Interview with Giampaolo Niccoli, MD, PhD, FESC
                                                    Interventional Cardiologist
                                                Assistant professor of Cardiology
                                                      Institute of Cardiology
                                       Catholic University of the Sacred Heart Rome, Italy


•   Angina.com: What are the main findings of the study?

•   Dr. Niccoli :In this study, we aimed at comparing coronary artery disease features at the time of a
    first acute coronary syndrome in type II diabetic and non-diabetic patients by coronary angiography
    and by optical coherence tomography.
•   In the angiographic substudy, diabetes was independently associated with both stenosis score and
    extent index. Furthermore, well developed collateral circulation toward the culprit vessel was more
    frequent in diabetic than in non-diabetic patients. In the optical coherence tomography substudy,
    at minimal lumen diameter site lipid quadrants were less and the lipid arc was smaller in diabetic
    than in non-diabetic patients. Furthermore, the most calcified cross-section along the culprit
    segment had a greater number of calcified quadrants and a wider calcified arc in diabetic than in
    non-diabetic patients. Superficial calcified nodules were more frequently found in diabetic than in
    non-diabetic patients.
•   Thus, in spite of potent pro-inflammatory, pro-oxidant and pro-thrombotic stimuli operating in type
    II diabetes, diabetic patients exhibit substantially more severe coronary atherosclerosis than non-
    diabetic patients at the time of a first acute coronary event. Better collateral development toward
    the culprit vessel, a predominantly calcific plaque phenotype and, probably, yet unknown
    protective factors operating in diabetic patients may explain these intriguing paradoxical findings.
Severity of coronary atherosclerosis in patients with a first acute coronary event: a diabetes paradox
                                Angina.com Interview with Giampaolo Niccoli, MD, PhD, FESC
                                                  Interventional Cardiologist
                                              Assistant professor of Cardiology
                                                    Institute of Cardiology
                                     Catholic University of the Sacred Heart Rome, Italy
                                                             (cont)

•   Angina.com: Were any of the findings unexpected?
•   Dr. Niccoli : When investigating the association of coronary artery disease burden
    and diabetes status in an acute setting, we found a more severe and extensive
    coronary atherosclerosis as compared to non- diabetic patients, although pro-
    thrombotic and pro-inflammatory stimuli are stronger in diabetic than non-
    diabetic patients.
•   Angina.com What should clinicians and patients take away from this study?
•   Dr. Niccoli : As demonstrated in our study, at the time of their first acute coronary
    syndrome, type II diabetic patients have more severe and extensive coronary
    atherosclerosis as compared to non-diabetic patients. Thus, diabetic patients may
    have protective factors that prevent a first acute coronary syndrome.
•   Indeed, we also showed that type II diabetic patients had better collateral
    circulation towards the culprit vessel than non-diabetic patients, Therefore,
    physicians should consider this apparent contradiction which is probably
    accounted for by the fact that diabetes offsets the detrimental effects of a high
    extent index on collateral development. Furthermore, a more calcific
    atherosclerosis may stabilize coronary plaque.
Severity of coronary atherosclerosis in patients with a first acute coronary event: a diabetes paradox
                                Angina.com Interview with Giampaolo Niccoli, MD, PhD, FESC
                                                  Interventional Cardiologist
                                              Assistant professor of Cardiology
                                                    Institute of Cardiology
                                     Catholic University of the Sacred Heart Rome, Italy
                                                             (cont)

•   Angina.com: What further research do you recommend as a result of your study?
•   Dr. Niccoli : Further studies will probably identify other, and possibly more
    important, causes of the paradox that in diabetic patients the first acute event
    occurs in the presence of more severe coronary artery disease. Interestingly,
    protective effect mediated by inhibition of matrix metalloproteinase-9, which
    reduces elastin degradation might operate in the coronary circulation and should
    be investigated.

•   Citation:
•   Severity of coronary atherosclerosis in patients with a first acute coronary event: a
    diabetes paradox.
•   Eur Heart J. 2013 Mar;34(10):729-41. doi: 10.1093/eurheartj/ehs393. Epub 2012
    Nov 27.
•   Niccoli G, Giubilato S, Di Vito L, Leo A, Cosentino N, Pitocco D, Marco V, Ghirlanda
    G, Prati F, Crea F.
•   Institute of Cardiology, Catholic University of the Sacred Heart, Largo F. Vito 1,
    00168 Rome, Italy.
Sexual Function After Left Ventricular Assist Device
                              Angina.com Author Interview: Peter Eckman, M.D.
                                            Department of Medicine
                        Cardiovascular Division, University of Minnesota Medical School,
                         420 Delaware Street S.E., MMC 508, Minneapolis, MN 55455



•   Angina.com: What are the main findings of the study?
•   Dr. Eckman: We found that 71% of men and 79% of women with a continuous flow
    left ventricular assist device (LVAD) met criteria for sexual dysfunction, as
    measured by the Changes in Sexual Function Questionnaire-14. Furthermore,
    more than half of the survey respondents noted worsening in arousal and orgasm.
    Age over 60 and history of ischemic heart disease were directly associated with
    sexual dysfunction. Psychological issues, including fear of damaging the LVAD, self-
    harm, device alarms, and hurting one’s partner were also cited, and most noted
    that the equipment was “somewhat” or a “major” obstacle to sexual satisfaction.
    Angina.com: Were any of the findings unexpected?
•   Dr. Eckman: LVADs have been shown to significant improve quality of life, so the
    relatively poorer sexual function was surprising. We were not surprised that older
    age and history of ischemic heart disease were associated with worse function.
    One participant said “Probably the least improved part of life before to after VAD.”
    On the other hand, we were encouraged to see that a notable minority of patients
    reported improvements in sexual function. We were also surprised that only a
    minority of patients reported improvement in desire.
Sexual Function After Left Ventricular Assist Device
                              Angina.com Author Interview: Peter Eckman, M.D.
                                            Department of Medicine
                        Cardiovascular Division, University of Minnesota Medical School,
                         420 Delaware Street S.E., MMC 508, Minneapolis, MN 55455
                                                     (cont)


•   Angina.com: What should clinicians and patients take away from this study?
•   Dr. Eckman: This data should inform subsequent studies designed to evaluate and
    improve understanding of the specific factors involved in sexual health after LVAD
    implant. Equipment was often cited as an obstacle, and improvements to external
    components may have a favorable impact on sexual function. Willingness to
    address concerns about sexual function may help minimize barriers to sexual
    activity and satisfaction after LVAD
•   Angina.com: What further research do you recommend as a result of your study?
•   Dr. Eckman: It is unclear if our findings were driven by physiologic changes such as
    altered blood flow and endothelial function, or any of the myriad other factors
    that impact sexual function. Study of the physiology of genital blood flow after
    implant of continuous-flow LVAD may provide additional insight into the relative
    importance of pathophysiology on sexual health after LVAD.
•   Citation:
•   Sexual Function After Left Ventricular Assist Device
•   Eckman PM, Dhungel V, Mandras S, et al.
    J Am Coll Cardiol. 2013;():.
    doi:10.1016/j.jacc.2013.02.022.
Yield and Diagnostic Value of Stress Myocardial Perfusion Imaging in Patients
                       Without Known Coronary Artery Disease Presenting with Syncope
                               Angina.com Interview with Wael A. AlJaroudi MD
                Division of Cardiovascular Disease American University of Beirut Medical Center
                                    Riad El-Solh, Beirut 1107 2020, Lebanon



• Angina.com: What are the main findings of the study?
• Answer: In this study, we investigated the diagnostic yield of stress
  myocardial perfusion imaging (MPI) for patients presenting for
  syncope without prior history of known coronary artery disease (CAD). Out
  of 700 patients, 659 (94%) had normal perfusion. Of the 41 patients with
  abnormal MPI, 18 had a coronary angiogram and half of them were falsely
  positive. The diagnostic yield of the stress MPI (95% CI) was 0.88% (0.3%,
  2.6%), 1.3%(0.3%, 4.0%) and 2.4% (0.61%, 7.3%) for patients with low,
  intermediate and high risk categories based on Framingham
  scores, respectively. Also, an abnormal stress MPI did not have prognostic
  value, in part due to the low numbers of abnormal studies and high false-
  positive results.
• As such, stress MPI for evaluation of syncope in patients without known
  CAD has a low diagnostic yield among all risk groups, and reaffirmation or
  revision of the appropriateness criteria should be considered.
Yield and Diagnostic Value of Stress Myocardial Perfusion Imaging in Patients
                       Without Known Coronary Artery Disease Presenting with Syncope
                               Angina.com Interview with Wael A. AlJaroudi MD
                Division of Cardiovascular Disease American University of Beirut Medical Center
                                    Riad El-Solh, Beirut 1107 2020, Lebanon
                                                      (cont)


• Angina.com: Were any of the findings unexpected?
• Answer: Based on our clinical experience, we were expecting a low
  diagnostic yield for stress MPI in such group of patients. However, there
  are limited data that justify using stress MPI to rule out ischemia as a
  cause of syncope in patients without known history of CAD. Given the high
  economic burden and radiation exposure from these tests, we sought to
  investigate whether indeed the diagnostic yield is high enough to justify
  using these test or whether the appropriateness criteria needs revisiting.
• Angina.com: What should clinicians and patients take away from this
  study?
• Answer: In patients without known CAD, syncope is not an anginal
  equivalent, and therefore the diagnostic yield of stress MPI is very low
  regardless of the Framingham score. Perhaps exercise stress
  echocardiogram might have better since it looks at structure, dynamic
  changes, and valves. This needs to be further studied and evaluated.
Yield and Diagnostic Value of Stress Myocardial Perfusion Imaging in Patients
                      Without Known Coronary Artery Disease Presenting with Syncope
                              Angina.com Interview with Wael A. AlJaroudi MD
               Division of Cardiovascular Disease American University of Beirut Medical Center
                                   Riad El-Solh, Beirut 1107 2020, Lebanon
                                                     (cont)


• Angina.com: What further research do you recommend as a result
  of your study?
• Answer: It would be interesting to evaluate the diagnostic yield of
  stress MPI for patients with known coronary artery disease and/or
  ischemic cardiomyopathy presenting with syncope.
• Citation:
• Yield and Diagnostic Value of Stress Myocardial Perfusion Imaging in
  Patients Without Known Coronary Artery Disease Presenting with
  Syncope
• Wael A. AlJaroudi, M. Chadi Alraies, Oussama Wazni, Manuel D.
  Cerqueira, and Wael A. Jaber
• Circ Cardiovasc Imaging. 2013;CIRCIMAGING.112.000051published
  online before print February 15 2013,
  doi:10.1161/CIRCIMAGING.112.000051
Renoprotective effect of remote ischemic postconditioning by intermittent balloon inflations
                            in patients undergoing percutaneous coronary intervention
                                            Angina.com Interview with
                               Spyridon Deftereos, MD; Georgios Giannopoulos, MD
                                        University of Athens, Athens, Greece



• Angina.com: What are the main findings of the study?
• Answer: The principal finding of this study is that a procedure of repeated
  inflations and deflations of the stent balloon used to perform PCI
  (percutaneous coronary intervention) in patients with non-ST-elevation
  myocardial infarction leads to a reduction in the periprocedural acute
  kidney injury rate. This effect seems to have a clinical impact, as it was
  accompanied by a reduction (of borderline statistical significance) in the
  30-day rate of death or rehospitalization for any cause.
• The physiological process underlying this procedure is termed “remote
  ischemic conditioning” (“remote” because the conditioning stimulus is
  applied to a site far away from the organ that is protected against tissue
  damage; in our case, the conditioning stimulus is applied to the heart and
  the conditioned organ is the kidney) and, although a lot of unanswered
  questions remain as to its mechanisms, experimental and clinical evidence
  has repeatedly demonstrated its existence.
Renoprotective effect of remote ischemic postconditioning by intermittent balloon inflations
                              in patients undergoing percutaneous coronary intervention
                                              Angina.com Interview with
                                 Spyridon Deftereos, MD; Georgios Giannopoulos, MD
                                          University of Athens, Athens, Greece
                                                          (cont)


•   Angina.com: Were any of the findings unexpected?
•   Answer: Actually no. The study results could have gone either way; that is a
    negative result would not have been a surprise either. However, we did have a
    smaller retrospective study published a couple of years ago suggesting that remote
    ischemic conditioning could be effective in this setting. Our present results show
    that that hypothesis-generating study pointed to the right direction.

•   Angina.com: What should clinicians and patients take away from this study?
•   Answer: An important point associated with this study is that cardiac
    interventionalists should always “remember the kidney” when performing cardiac
    catheterizations, especially in patients with acute coronary syndromes (since these
    patients are more vulnerable to acute renal damage compared to those
    undergoing elective procedures). Effective measures of renoprotection, including
    appropriate hydration, withholding of nephrotoxic drugs, parsimonious use of
    contrast and, probably (if our results are confirmed by further clinical trials),
    remote ischemic conditioning should be employed to reduce the likelihood of
    renal damage.
Renoprotective effect of remote ischemic postconditioning by intermittent balloon inflations
                              in patients undergoing percutaneous coronary intervention
                                              Angina.com Interview with
                                 Spyridon Deftereos, MD; Georgios Giannopoulos, MD
                                          University of Athens, Athens, Greece
                                                          (cont)


•   Angina.com: What further research do you recommend as a result of your study?
•   Answer: Our positive results, along with other recent ones, should prompt a larger
    clinical study, powered enough to demonstrate clinical efficacy, if any, in terms of
    hard clinical endpoints.
•   Our study was underpowered to have a primary clinical endpoint, but the
    secondary finding of a reduction in the 30-day clinical event rate is certainly
    encouraging.
•   Citation:
•   Renoprotective effect of remote ischemic postconditioning by intermittent balloon
    inflations in patients undergoing percutaneous coronary intervention
•   Deftereos S, Giannopoulos G, Tzalamouras V, Raisakis K, Kossyvakis C, Kaoukis A,
    Panagopoulou V, Karageorgiou S, Avramides D, Toutouzas K, Hahalis G, Pyrgakis V,
    Manolis AS, Alexopoulos D, Stefanadis C, Cleman MW.
•   Department of Cardiology, Athens General Hospital “G. Gennimatas”, Athens,
    Greece.
•   J Am Coll Cardiol. 2013 Mar 13. pii: S0735-1097(13)01001-2. doi:
    10.1016/j.jacc.2013.02.023. [Epub ahead of print]
Duration of Ventilations during Cardiopulmonary Resuscitation by Lay-Rescuers and First Responders: Relationship between Delivering
                                                    Chest Compressions and Outcomes
                                               Angina.com Interview with S.G. Beesems, MSc
                                                            afdeling Cardiologie
                                                               Kamer G4-248
                                             Academisch Medisch Centrum 1105 AZ Amsterdam


•      Angina.com: What are the main findings of the study?
•      Answer: The results of our study show that lay-rescuers require a median ventilation time of 7
       seconds to complete two ventilation attempts. Only 21% are able to fully meet the ventilation
       guidelines of 2010. However, 97% of all rescuers provided chest compressions above the
       recommended minimum of 60 chest compressions in one minute, 88% of all administered over 70
       chest compressions in one minute and 63% of all administered over 80 chest compressions in one
       minute. Hospital survival of patients with all rhythms was 25%. Therefore, the objective of
       delivering sufficient chest compressions is met by all but a few rescuers.


•      Angina.com: Were any of the findings unexpected?
•      Answer: In our study, better survival was observed with longer ventilations. This is against the
       findings of other studies with long interruptions of chest compressions, mainly caused by pauses
       associated with defibrillation shocks. This paradox can be attributed to the fact that rescuers who
       had the longest pauses for ventilations also arrived and shocked earlier with the AED. After
       adjustment for the baseline factors, longer pauses for ventilation were not associated with worse
       survival. It is also possible that the suggested detrimental effect of perishock pauses does not apply
       to pauses for ventilation
Duration of Ventilations during Cardiopulmonary Resuscitation by Lay-Rescuers and First Responders: Relationship between Delivering
                                                Chest Compressions and Outcomes
                                           Angina.com Interview with S.G. Beesems, MSc
                                                        afdeling Cardiologie
                                                           Kamer G4-248
                                         Academisch Medisch Centrum 1105 AZ Amsterdam
                                                               (cont)


• Angina.com: What should clinicians and patients take away from
  this study?
• Answer: Lay-rescuers with CPR training that includes chest
  compressions and ventilations are able to perform CPR better than
  previously believed. Longer pauses for ventilations are not
  associated with worse outcome. Guidelines may allow longer
  pauses for ventilations without detriment to survival.
• Citation:
• Duration of Ventilations during Cardiopulmonary Resuscitation by
  Lay-Rescuers and First Responders: Relationship between Delivering
  Chest Compressions and Outcomes.
• Beesems SG, Wijmans L, Tijssen JG, Koster RW.
• 1Academic Medical Center, Amsterdam, The Netherlands.
  Circulation. 2013 Mar 18. [Epub ahead of print]
Troponin I, but not BNP, is Associated with Phosphorus, Calcium and Vitamin D in Stable Coronary Artery Disease
                                   Angina.com Interview with José Pedro L. Nunes M.D.
                                    Associate professor at University of Porto, Portugal.




• Angina.com: What are the main findings of the study?
• Dr. Nunes: Elevated plasma cardiac troponin, elevated plasma phosphorus
  and decreased plasma vitamin D are known to be associated with negative
  outcomes. In this study, data was obtained from a relatively small number
  of patients (sixty) with stable coronary artery disease and preserved left
  ventricular function, and patients with higher values for cardiac troponin I
  were shown to have higher plasma phosphorus, higher plasma calcium
  and lower plasma 25-OH vitamin D values when compared to patients
  with lower values for troponin. The same association was not found with
  BNP.
•
  Angina.com: Were any of the findings unexpected?
• Dr. Nunes: When you start an investigation, you do not know what you will
  find, but previous studies, carried out in renal patients, had shown an
  association between troponin levels and FGF-23, which itself is associated
  to phosphorus.
Troponin I, but not BNP, is Associated with Phosphorus, Calcium and Vitamin D in Stable Coronary Artery Disease
                                    Angina.com Interview with José Pedro L. Nunes M.D.
                                     Associate professor at University of Porto, Portugal.
                                                            (cont)



•   Angina.com: What should clinicians and patients take away from this study?
•   Dr. Nunes: Given the relatively small number of patients under study, the conclusions should be
    seen with some caution. In any event, troponin is no longer seen solely as a marker for myocardial
    infarction. Cardiac/renal connections could find here yet another important field of future research.
    As stated in the text, “the hypothesis is raised that deleterious effects of higher phosphorus and/or
    calcium and of lower vitamin D levels on the heart are reflected by an increase in plasma troponin
    I”.
•   Angina.com: What recommendations do you have for future research as a result of your study?
•   Dr. Nunes: It would be important to try to confirm these findings in larger and different sets of
    patients. If confirmed, these findings could be a starting point for significant advances in the
    understanding of mechanisms of cardiac injury, perhaps involving diet, lack of sunlight exposure
    and with a possible impact on vitamin D supplementation.
•   Citation:
•   Troponin I, but not BNP, is Associated with Phosphorus, Calcium and Vitamin D in Stable Coronary
    Artery Disease
•   Nunes J.P.L.
•   Faculdade de Medicina da Universidade do Porto: Center for Research in Health Technologies and
    Information Systems (Cintesis), Porto
•   Kidney Blood Press Res 2013;37:43-47
    (DOI:10.1159/000343399)
•   Published online first (Issue-in-Progress)
Primary Percutaneous Coronary Intervention in Patients With Acute Myocardial Infarction, Resuscitated Cardiac Arrest, and Cardiogenic
                                     Shock : The Role of Primary Multivessel Revascularization
                                      Angina.com Interview with Darren Mylotte MB BCh MD
                                 ICPS – Institut Cardiovasculaire Paris Sud · Interventional Cardiology




• Angina.com: What are the main findings of the study?
• Dr. Mylotte: The study demonstrates the very high
  mortality rate of patients with STEMI, cardiogenic shock
  and resuscitated cardiac arrest: 65.4% at 6-months. It also
  demonstrates that those with more extensive myocardial
  ischemia due to more widespread coronary disease have
  worse outcomes compared to those with less ischemia.
  Furthermore, it demonstrates that in those with
  multivessel coronary disease, multivessel PCI appears to
  improve survival compared to culprit-only PCI. Finally,
  successful PCI and multivessel PCI were independent
  predictors of 6-month survival in those with multivessel
  disease.
Primary Percutaneous Coronary Intervention in Patients With Acute Myocardial Infarction, Resuscitated Cardiac Arrest, and Cardiogenic
                                     Shock : The Role of Primary Multivessel Revascularization
                                      Angina.com Interview with Darren Mylotte MB BCh MD
                                 ICPS – Institut Cardiovasculaire Paris Sud · Interventional Cardiology
                                                                 (cont)


• Angina.com: Were any of the findings unexpected?
• Dr. Mylotte: The fact that multivessel primary PCI was associated
  with improved survival was not a surprise. Indeed the study
  hypothesis stemmed from other data suggesting enhanced
  outcomes in this critically-ill patient population. However, the
  magnitude of the survival advantage (absolute survival advantage
  23.5%) was unexpected. While this may also reflect some selection
  or treatment bias, there is a clear signal that in selected cases
  multivessel PCI is advantageous.
• Angina.com: What should clinicians and patients take away from
  this study?
• Dr. Mylotte: I think that the central message of this study is that in
  patients with multivessel disease and cardiogenic shock, refractory
  to culprit-artery PCI, that further revascularization should be
  considered.
Primary Percutaneous Coronary Intervention in Patients With Acute Myocardial Infarction, Resuscitated Cardiac Arrest, and Cardiogenic
                                         Shock : The Role of Primary Multivessel Revascularization
                                          Angina.com Interview with Darren Mylotte MB BCh MD
                                     ICPS – Institut Cardiovasculaire Paris Sud · Interventional Cardiology
                                                                     (cont)


•       Angina.com: What further research do you recommend as a result of your
        study?
•       Dr. Mylotte: I think that this study highlights the need for further randomized trials
        comparing revascularization strategies in this very high-risk group of patients.
        Furthermore, it demonstrates the limitations of coronary revascularization and
        that other treatments need to be developed in order to significantly improve
        outcomes.
•       Citation:
•       Primary percutaneous coronary intervention in patients with acute myocardial
        infarction, resuscitated cardiac arrest, and cardiogenic shock: the role of primary
        multivessel revascularization.
•       Mylotte D, Morice MC, Eltchaninoff H, Garot J, Louvard Y, Lefèvre T, Garot P.
•       Institut Cardiovasculaire Paris Sud, Institut Hospitalier Jacques Cartier, Massy, and
        Hôpital Claude Galien, Quincy, France.
        JACC Cardiovasc Interv. 2013 Feb;6(2):115-25. doi: 10.1016/j.jcin.2012.10.006.
        Epub 2013 Jan 23.
Genetic Determinants of Dabigatran Plasma Levels and Their Relation to Bleeding.
                                    Angina.com Interview with Guillaume Paré MD, MSc, FRCPC
                                   Assistant ProfessorPathology and Molecular Medicine (primary)
                                                 Clinical Epidemiology & Biostatistics
                                    Canada Research Chair in Genetic and Molecular Epidemiology
       Director, Genetic and Molecular Epidemiology Laboratory Visiting Scientist, Harvard Medical School McMaster University


•   Angina.com: What are the main findings of the study?

•   Dr. Pare: We conducted a genome-wide association study of dabigatran peak and trough
    concentration in 2,944 RE-LY participants with both genetic and pharmacokinetic data. The CES1
    SNP rs2244613 was associated with a 15% decrease in trough concentrations per allele, and the
    ABCB1 SNP rs4148738 and CES1 SNP rs8192935 were associated with peak concentrations at
    genome-wide significance (P<9 x 10-8), also with a gene-dose effect. CES1 encodes for the liver
    carboxylesterase 1 enzyme, an esterase responsible for the biotransformation of dabigatran
    etexilate into the active metabolite, dabigatran. The ABCB1 gene encodes for P-glycoprotein, an
    ATP-dependent drug efflux pump for xenobiotic compounds with broad substrate
    specificity. Dabigatran etexilate, but not dabigatran, is an ABCB1 substrate and ABCB1 inhibitors
    increase dabigatran bioavailability by 10-20%. When tested for association with bleed, the CES1
    SNP rs2244613 was associated with a lower risk of any bleeding (OR=0.67, 95%CI 0.55-0.82; P=7 x
    10-5) in dabigatran-treated participants, and with a consistent but non-significant lower risk of
    major bleeding (OR=0.66, 95%CI 0.43-1.01). Importantly, the interaction between treatment
    (warfarin versus all dabigatran) and carrier status was statistically significant (P=0.002) with carriers
    having less bleeding with dabigatran than warfarin (HR=0.59, 95%CI 0.46-0.76; P=5.2 x 10-5) in
    contrast to no difference in noncarriers (HR=0.96, 95%CI 0.81-1.14; P=0.65).
Genetic Determinants of Dabigatran Plasma Levels and Their Relation to Bleeding.
                                   Angina.com Interview with Guillaume Paré MD, MSc, FRCPC
                                  Assistant ProfessorPathology and Molecular Medicine (primary)
                                                Clinical Epidemiology & Biostatistics
                                   Canada Research Chair in Genetic and Molecular Epidemiology
      Director, Genetic and Molecular Epidemiology Laboratory Visiting Scientist, Harvard Medical School McMaster University
                                                               (cont)

•   Angina.com: Were any of the findings unexpected?
•   Dr. Pare: While the two gene loci identified were to be expected given dabigatran
    etexilate’s metabolism, we were surprised by the strength of the genetic effects,
    especially on bleeds. Also, dabigatran etexilate was thought to be converted into
    dabigatran completely and immediately by esterases. Identification of a genetic
    variant of CES1 associated with drug exposure suggests that biotransformation into
    dabigatran could be a rate-limiting factor in certain individuals.
•   Angina.com: What should clinicians and patients take away from this study?
•   Dr. Pare: These results are significant as the relative risk of bleeding was 0.73
    (95%CI 0.63-0.86) for the 32.8% of RE-LY participants that are rs2244613 minor
    allele carriers versus noncarriers, whereas the reported relative risk of bleeding
    was 0.86 (95% CI 0.81-0.93) for the lower (110 mg bid) versus higher dose (150 mg
    bid) of dabigatran in the overall study. The observed genetic effect was thus larger
    than the effect of drug dosage in the parent study, suggesting a potential role of
    genetics to help determine the optimal dose of dabigatran etexilate in individual
    patients. For example, carriers of the protective allele could perhaps be safely
    prescribed the higher dose of dabigatran etexilate even if elderly.
Genetic Determinants of Dabigatran Plasma Levels and Their Relation to Bleeding.
                                    Angina.com Interview with Guillaume Paré MD, MSc, FRCPC
                                   Assistant ProfessorPathology and Molecular Medicine (primary)
                                                 Clinical Epidemiology & Biostatistics
                                    Canada Research Chair in Genetic and Molecular Epidemiology
       Director, Genetic and Molecular Epidemiology Laboratory Visiting Scientist, Harvard Medical School McMaster University
                                                                (cont)

•   Angina.com: What further research do you recommend as a result of your study?
•   Dr. Pare: Many key questions remain to be addressed. Among them is whether the CES1
    SNP rs2244613 is associated with ischemic events as well as bleeds. Indeed, despite the large
    sample size, only 66 dabigatran etexilate-treated individuals suffered from any ischemic event in
    the genetic analysis sub-group of the RE-LY trial and power to detect a genetic association was
    accordingly limited. Larger studies will therefore be necessary to properly answer this
    question. Finally, clinical utility of CES1 genotyping will need to be assessed through rigorous
    studies carefully evaluating the health benefit and cost of incorporating genotypes into dose
    selection decisions.

•   Citation:
•   Genetic Determinants of Dabigatran Plasma Levels and Their Relation to Bleeding.
•   Paré G, Eriksson N, Lehr T, Connolly S, Eikelboom J, Ezekowitz MD, Axelsson T, Haertter S, Oldgren J,
    Reilly P, Siegbahn A,
•   Syvänen AC, Wadelius C, Wadelius M, Zimdahl-Gelling H, Yusuf S, Wallentin L.
    1Population Health Research Institute, Hamilton Health Sciences and McMaster University,
    Hamilton, Canada.
    Circulation. 2013 Mar 6. [Epub ahead of print]
Rivaroxaban Reduces Spontaneous and Large Myocardial Infarctions: Findings from the ATLAS ACS 2 – TIMI 51 Trial
                                 Angina.com Interview with Matthew A. Cavender, M.D.
                                           Research Fellow, TIMI Study Group
                                            Brigham and Women’s Hospital
                                                Harvard Medical School
                                             Boston, Massachusetts 02115



• Angina.com: What are the main findings of the study?
• Dr. Cavender: The ATLAS ACS 2 – TIMI 51 trial showed that
  treatment with rivaroxaban reduces cardiovascular death,
  myocardial infarction and stroke. To better understand how this
  drug improves cardiovascular outcomes, it was important to
  understand the effect of the rivaroxaban on specific types of
  myocardial infarctions.

• Angina.com: Were any of the findings unexpected?
• Dr. Cavender: In the overall trial, rivaroxaban 2.5 mg twice daily
  reduced cardiovascular death, myocardial infarction, or stroke. In
  addition, rivaroxaban 2.5 mg was shown to reduce cardiovascular
  death. The present study offers insight into the possible
  mechanisms leading to this mortality benefit since rivaroxaban
  reduces both spontaneous and large myocardial infarctions.
Rivaroxaban Reduces Spontaneous and Large Myocardial Infarctions: Findings from the ATLAS ACS 2 – TIMI 51 Trial
                                Angina.com Interview with Matthew A. Cavender, M.D.
                                          Research Fellow, TIMI Study Group
                                           Brigham and Women’s Hospital
                                               Harvard Medical School
                                            Boston, Massachusetts 02115
                                                        (cont)


• Angina.com: What should clinicians and patients take away from
  this study?
• Dr. Cavender: Our analysis showed the majority of myocardial
  infarctions in the ATLAS ACS 2 – TIMI 51 trial occurred
  spontaneously due to plaque rupture and coronary artery
  thrombosis. Rivaroxaban was given to patients to prevent these
  types of thrombotic events. We found that treatment with
  rivaroxaban significantly reduced spontaneous myocardial
  infarctions by approximately 20%, and both the 2.5 and 5 mg doses
  had directionally similar effects.
• Citation:
• Presented at 2013 ACC American College of Cardiology March 2013
  Rivaroxaban Reduces Spontaneous and Large Myocardial
  Infarctions:
  Findings from the ATLAS ACS 2 – TIMI 51 Trial
Value of Myocardial Perfusion Imaging in Renal Transplant Evaluation
                                Angina.com Interview with: Dr. Chong Ghee Chew
      Department of Nuclear Medicine, PET and Bone Mineral Densitometry, Royal Adelaide Hospital, Adelaide, SA




• Angina.com: What are the main findings of the study?
• Answer: This is a retrospective audit of the cardiac outcomes of renal
  failure patients who had been transplanted in South Australia between
  1999 to 2009, who had myocardial perfusion SPECT scan for the transplant
  assessment. The results represent ”real world” outcomes as the scans
  were performed in the 3 major teaching hospitals in SA. 2 endpoints –
  “soft” = inpatient care with angina +/- PCI +/- CABG, and “hard” =
  inpatient care with myocardial infarction or cardiac death. With a negative
  scan this cohort had a statistically significant lower soft endpoint event
  rate than a positive scan …3.9% vs 20.8%, hazard ratio of 4.4 at 5 years
  post scan. The hard endpoint event rate was also lower for those with a
  negative scan but the difference did not reach statistical significance. The
  event rates of hard and soft endpoints were no different for the negative
  scans that were performed with a tachycardic stress (treadmill exercise,
  dobutamine or external wire right atrial pacing) versus dipyridamole
  induced coronary vasodilatation.
Value of Myocardial Perfusion Imaging in Renal Transplant Evaluation
                                Angina.com Interview with: Dr. Chong Ghee Chew
      Department of Nuclear Medicine, PET and Bone Mineral Densitometry, Royal Adelaide Hospital, Adelaide, SA

                                                       (cont)


• Angina.com: Were any of the findings unexpected?
• Answer: No
• Angina.com: What should clinicians and patients take away from this
  study?
• Answer: Myocardial perfusion SPECT scan is a good predictor of cardiac
  events in renal failure patients who are being considered for
  transplantation.
• This is a valid test for transplant assessment.
• Angina.com: What further research do you recommend as a result of
  your study?
• The study did not include patients who were assessed but were not
  transplanted. We are planning another similar audit to look at this cohort.
• Citation:
• ACC 2013 American College Cardiology Presentation Spring 2013
Angiographic and clinical characteristics of type 1 versus type 2 perioperative myocardial infarction
                                     Angina.com Interview with: Ivan Hanson, M.D.
                                         Chief Interventional Cardiology Fellow
                                           Beaumont Hospital, Royal Oak, MI



•   Angina.com: What are the main findings of the study?
•   Dr. Hanson: Over half of patients with perioperative myocardial infarction (PMI)
    have invasive angiographic evidence of plaque rupture (type 1 PMI). Patients with
    type 1 PMI had more intraoperative hypotension, higher incidence of transmural
    ischemia, larger infarct size and trended toward worse outcome.

•   Angina.com: Were any of the findings unexpected?
•   Dr. Hanson: Our hypothesis was that the majority of PMI events were due to
    hemodynamic insults in the setting of stable, but severely stenotic, coronary artery
    stenoses (type 2 PMI).
•   Not only did we observe a higher incidence of type 1 PMI than type 2 PMI, but
    type 1 PMI patients actually had greater incidence of intraoperative hypotension.
    This challenges the notion that hemodynamic pertubations are more closely
    associated with type 2 PMI.
•   Furthermore, clinical assessment alone, including knowledge of intraoperative
    hemodynamics and postoperative ECG findings, are insufficient to diagnose type of
    PMI.
Angiographic and clinical characteristics of type 1 versus type 2 perioperative myocardial infarction
                                     Angina.com Interview with: Ivan Hanson, M.D.
                                         Chief Interventional Cardiology Fellow
                                           Beaumont Hospital, Royal Oak, MI
                                                          (cont)


•   Angina.com: What should clinicians and patients take away from this study?
•   Dr. Hanson: Our findings suggest that if a sophisticated understanding of type of
    PMI is desired, invasive angiography may be useful.
•   Patients that were formally assumed to have type 2 PMI may in fact have plaque
    rupture and coronary thrombosis, which would not be expected to resolve by
    optimization of coronary blood flow alone.
•   Angina.com: What further research do you recommend as a result of your study?
•   Dr. Hanson: To test the hypothesis that tailoring management of PMI based on
    angiographic findings results in optimal outcomes, a much larger study is required.
•   I am also very interested in the invasive imaging characteristics of plaques that
    lead to, or have caused, PMI.

•   Citation:
•   Angiographic and clinical characteristics of type 1 versus type 2 perioperative
    myocardial infarction.
Centralized, Stepped, Patient Preference–Based Treatment for Patients With Post–Acute Coronary Syndrome Depression
                                         Angina.com Interview with Karina W. Davidson, PhD
                                Center for Behavioral Cardiovascular Health, Department of Medicine,
                                      Columbia University Medical Center, New York, New York



•   Angina.com: What are the main findings of the study?

•   Dr. Davidson: With a grant from the National Institutes of Health’s National Heart, Lung, and Blood
    Institute (NHLBI), Karina W. Davidson, PhD, director of the Center for Behavioral Cardiovascular
    Health at Columbia University Medical Center (CUMC), together with her colleagues at multiple
    sites in the U.S., completed a randomized controlled trial with 150 patients with elevated
    depressive symptoms two to six months after their hospitalization for heart disease. Patients were
    recruited from seven centers across the United States and were randomized to receive either six
    months of their preferred depression care (therapy, antidepressants or both) or to receive
    depression care determined by their physician upon notification that the patient had elevated
    depressive symptoms.

•   In the self-selected group, patients were given a choice of whether they wanted therapy or
    antidepressants, or both. “We tried to help them really think through what does it take to take a pill
    every day; what does it take to talk to a person about painful feelings and do homework, and really
    consider what will fit into their life,” said Dr. Davidson. “And every six to eight weeks, we revisited
    how their chosen treatment was working for them, and helped them make adjustments based
    upon their feedback.”
Centralized, Stepped, Patient Preference–Based Treatment for Patients With Post–Acute Coronary Syndrome Depression
                                         Angina.com Interview with Karina W. Davidson, PhD
                                Center for Behavioral Cardiovascular Health, Department of Medicine,
                                      Columbia University Medical Center, New York, New York
                                                               (cont)


•   Therapy in this study was given virtually (by phone or web-based) to ensure that patients had access to clinicians
    of a similar skill set – so two counselors (one at CUMC and another at Yale) provided the therapy in this study. The
    virtual therapy also helped connect patients with therapists at times that were most convenient for them. “Maybe
    8 p.m. at your home is the right time for you, but not 8 p.m. if you have to go for a clinic or hospital and wait half
    an hour and travel back home afterwards, etc.,” said Dr. Davidson, who is also professor of behavioral medicine in
    medicine, cardiology, and psychiatry, CUMC. The therapy used in the trial was problem solving treatment (PST) – a
    practical, hands-on approach that asks patients to choose the problems in their life that they want to tackle,
    teaches them how to problem solve, and then they go out and do homework to try the solutions and come back
    and talk about what worked, what didn’t, and tackle the next problem.

•   Findings showed that depressive symptoms were reduced more in the patient-selected, centralized depression
    care group, than in the patients receiving usual care. At the start of the trial, patients had an average of 19 points
    on the BDI depression scale – at the end of the trial, BDI scores in the patient-selected group were significantly
    reduced by an average of 3-6 points more than the reduction that happened in the usual care group. Getting
    down below 10 on the BDI puts one in the normal range – and almost half of the patients who received patient-
    selected, centralized treatment get down to the normal range; whereas only one-third in the usual care group got
    down to the normal range.

•   Importantly, researchers found that running centralized depression care at seven sites nationwide was effective,
    suggesting that it is feasible to deliver depression care to those who don’t have specialists in their area. Overall,
    the total health care costs for the self-selected group was $325 less than the control group, even when factoring in
    the cost of providing them with the therapy. The centralized depression care group demonstrated a trend towards
    fewer hospitalizations, compared to the usual care group.
Centralized, Stepped, Patient Preference–Based Treatment for Patients With Post–Acute Coronary Syndrome Depression
                                         Angina.com Interview with Karina W. Davidson, PhD
                                Center for Behavioral Cardiovascular Health, Department of Medicine,
                                      Columbia University Medical Center, New York, New York
                                                               (cont)


•   Angina.com: Were any of the findings unexpected?
•   Dr. Davidson: Most depression trials in this area have gotten effect size of .3, which is one-third of
    the standard deviation. And we got .6, or 2/3 of the standard deviation. So we had almost double
    the size of treatment improvement compared to other people.
•   The research team believes that the difference in their findings can be explained in the difference
    between how previous published trials were conducted, versus how their trial was conducted. In
    previous trials, patients were often offered either one type of treatment or usual care, and they
    had to remain with that selection for the remainder of the trial. However, because it’s well known
    that there isn’t one magic bullet for depression; one thing doesn’t work for all patients, the
    CODIACS Vanguard trial was designed with a flexible algorithm that allowed the clinical team to
    regularly liaise with the patients to help them keep different treatment options and combinations,
    until one was found that suited them best.
•   Angina.com: What should clinicians and patients take away from this study?
•   Dr. Davidson: About 1.2 million Americans survive an ACS event every year and many of them have
    clinically significant and persistent depression. Post-ACS depression is also associated with an
    increased risk of ACS recurrence and with an increase in the relative risk of all-cause mortality, the
    authors write in the study background. These findings show that treating heart attack survivors for
    depressive symptoms is feasible, effective, and even has the tantalizing possibility of being cost-
    neutral. Importantly, treating depressive symptoms—as opposed to a diagnosis of depression—
    should be determined by an alliance between a healthcare professional and the patient – and
    regularly assessed for effectiveness and altered to continue to meet the patient’s needs.
Centralized, Stepped, Patient Preference–Based Treatment for Patients With Post–Acute Coronary Syndrome Depression
                                        Angina.com Interview with Karina W. Davidson, PhD
                               Center for Behavioral Cardiovascular Health, Department of Medicine,
                                     Columbia University Medical Center, New York, New York
                                                              (cont)


•   Angina.com: What further research do you recommend as a result of your
    study?Dr. Davidson: Dr. Karina Davidson and her team plan to explore whether
    these results can be replicated in a much larger nationwide randomized controlled
    trial, to examine whether treating them for depressive symptoms lowers their risk
    of death or having another heart attack.
•   Citation:
•   Centralized, Stepped, Patient Preference–Based Treatment for Patients With Post–
    Acute Coronary Syndrome Depression: CODIACS Vanguard Randomized Controlled
    Trial ONLINE FIRST
•   Karina W. Davidson, PhD; J. Thomas Bigger, MD; Matthew M. Burg, PhD; Robert M.
    Carney, PhD; William F. Chaplin, PhD; Susan Czajkowski, PhD; Ellen Dornelas, PhD;
    Joan Duer-Hefele, RN; Nancy Frasure-Smith, PhD; Kenneth E. Freedland, PhD;
    Donald C. Haas, MD, MPH; Allan S. Jaffe, MD; Joseph A. Ladapo, MD, PhD; Francois
    Lespérance, MD; Vivian Medina, MSW; Jonathan D. Newman, MD, MPH; Gabrielle
    A. Osorio, BS; Faith Parsons, BS; Joseph E. Schwartz, PhD; Jonathan A. Shaffer, PhD;
    Peter A. Shapiro, MD; David S. Sheps, MD; Viola Vaccarino, MD, PhD; William
    Whang, MD, MS; Siqin Ye, MD
•   JAMA Intern Med. 2013;():1-8. doi:10.1001/jamainternmed.2013.915.
    doi:10:1001/jama.2010.920
Direct-to-consumer television advertising exposure, diagnosis with high cholesterol, and statin use
                                Angina.com: Interview with Dr. Jeff Niederdeppe, Ph.D.
                                                   Assistant Professor
                                   Department of Communication, Cornell University
                                                          (cont)


•   Angina.com: What are the main findings of the study?
•   Dr. Niederdeppe: We examined the relationship between exposure to direct-to-consumer
    advertising (DTCA) for cholesterol-lowering drugs (most of which are statins) and two clinical
    variables: diagnosis with high cholesterol, and taking a statin drug in the past year. We found that
    greater exposure to statin DTCA was associated with an increased likelihood of being diagnosed
    with high cholesterol and taking a statin to treat it. We also found that these relationships were
    strongest for adults at comparatively low risk for cardiac events – those who tended to be younger,
    did not have high blood pressure, and did not smoke regularly. We did not find positive
    relationships between statin DTCA exposure and high cholesterol diagnosis or statin use for those
    at higher levels of risk, or those who reported having a history of heart disease or a previous heart
    attack.

•   Angina.com: Were any of the findings unexpected?
•   Dr. Niederdeppe: We were not surprised that statin DTCA exposure would be associated with high
    cholesterol diagnosis and statin use among lower-risk people, since that pattern has been found
    before for other types of pharmaceutical drugs. We were surprised, however, at the lack of a
    positive relationship for those at higher risk. In fact, we even found a negative relationship between
    DTCA exposure and statin use among high-risk women (those with a history of heart disease, a
    previous heart attack, and/or diabetes). In other words, greater exposure to statin DTCAs was linked
    to a lower likelihood of using the drug for this group. These results are puzzling and should be
    considered preliminary, but we think that future work should see whether or not this pattern exists
    in other studies with other populations.
Direct-to-consumer television advertising exposure, diagnosis with high cholesterol, and statin use
                                 Angina.com: Interview with Dr. Jeff Niederdeppe, Ph.D.
                                                    Assistant Professor
                                    Department of Communication, Cornell University



•   Angina.com: What should clinicians and patients take away from this study?
•   Dr. Niederdeppe: I’ll first acknowledge that I am not an M.D., and that I understand each patient has a unique set
    of risk factors, personal and family history, and medical profile. That said, one takeaway point is that we can
    expect patients to learn about pharmaceutical drugs from DTCA, and that is going to influence the clinical
    encounter. The average American is exposed to 16 hours of DTCA every year; in contrast they see the doctor an
    average of 15 minutes per year. Clinicians should be prepared to discuss both the risks and benefits of heavily-
    advertised pharmaceutical drugs.
•   I would encourage patients to ask their doctor about the ratio of risks to benefits related to drugs they see
    advertised. While all DTCAs have to disclose drug risks, the ads tend to do this in a way that focuses greater
    attention on benefits than risks.

•   Angina.com: What further research do you recommend as a result of your study?
•   Dr. Niederdeppe: Many statin DTCAs contain information about lifestyle changes (which are always recommended
    first to reduce cholesterol) and pharmaceutical drugs. One important question is how these ads might influence
    people’s lifestyle decisions – whether or not to engage in regular exercise, eat a heart-healthy diet, and maintain a
    healthy weight. We don’t yet know enough about some of the secondary impacts that these ads may be having on
    factors that influence the public’s health.
•   Citation:
•   Direct-To-Consumer Television Advertising Exposure, Diagnosis with High Cholesterol, and Statin Use.
•   Niederdeppe J, Byrne S, Avery RJ, Cantor J.
•   Department of Communication, Cornell University, 328 Kennedy Hall, Ithaca, NY, 14853, USA
    J Gen Intern Med. 2013 Mar 6. [Epub ahead of print
Impact of Socioeconomic Deprivation and Area of Residence on Access to Coronary Revascularization and Mortality After a First Acute
                                                      Myocardial Infarction in Québec
                                            Angina.com Interview with Cardiology Researcher:
                                                     Stéphane Rinfret MD, SM, FRCPC
                    Institut universitaire de cardiologie et de pneumologie de Québec, Québec City, Québec, Canada




• Angina.com: What are the main findings of the study?
• Dr. Rinfret : Study shows that patients unfortunately do not
  persist with dual antiplatelet therapy as we like to think
  they would do because of the consequences. A simple
  telephone intervention clearly helped to reach almost
  perfect adherence.

• Angina.com: Were any of the findings unexpected?
• Dr. Rinfret : Self-reported adherence is worth nothing. We
  relied on pharmacy data. When we compared with the
  answers patients gave to interviewers at 6 months and 1
  year, there was very low correlation. Therefore pharmacy
  data are absolutely required to assess true adherence.
Impact of Socioeconomic Deprivation and Area of Residence on Access to Coronary Revascularization and Mortality After a First Acute
                                                         Myocardial Infarction in Québec
                                               Angina.com Interview with Cardiology Researcher:
                                                        Stéphane Rinfret MD, SM, FRCPC
                       Institut universitaire de cardiologie et de pneumologie de Québec, Québec City, Québec, Canada



•      Angina.com: What should clinicians and patients take away from this study?
•      Dr. Rinfret : We need to take responsibility and put systems in place to follow how
       patients take their medication following stenting.
       A simple advise not to stop the medication is clearly not enough.
•      Angina.com: What further research do you recommend as a result of your study?
•      Dr. Rinfret : As newer DES will likely mandate a shorter DAT duration, a shorter
       intervention with 1 or 2 calls may be enough.
       This will have to be tested.
•      Citation:
•      Impact of Socioeconomic Deprivation and Area of Residence on Access to
       Coronary Revascularization and Mortality After a First Acute Myocardial
       Infarction in Québec
•      Claudia Blais, PhD, Denis Hamel, MSc, Stéphane Rinfret, MD, SM, FRCPC
•      Impact of Socioeconomic Deprivation and Area of Residence on Access to
       Coronary Revascularization and Mortality After a First Acute Myocardial Infarction
       in Québec
•      Canadian Journal of Cardiology, Volume 28, Issue 2, March–April 2012,
       Pages 169–177
Comparison of Coronary CT Angiography Image Quality With and Without Breast Shields.
                             Angina.com Interview with Dr. Eddie Hulten, MD MPH
                                     Non-Invasive Cardiovascular Imaging
                                        Brigham and Women’s Hospital
                                            Harvard Medical School



• Angina.com: What are the main findings of the study?
• Dr. Hulten: We reviewed a convenience sample of patients who had
  previously been scanned with bismuth breast shields while undergoing
  coronary CT angiography. We found differences in noise and an
  association of different plaque types by shielded versus non-shielded
  groups. Image quality graded on a Likert scale was not significantly
  different with no difference in uninterpretable segments according to
  shielded versus not shielded.

• Angina.com: Were any of the findings unexpected?
• Dr. Hulten: We expected the difference in noise to be more dramatic and
  the image quality to be poorer for the shielded patients. However, it is
  important to note that most of the scans were performed using
  retrospective ECG gating, which delivers a higher overall radiation than
  most contemporary scans. Thus, the results should not be extrapolated to
  patients scanned using prospectively-triggered ECG gating or high pitch
  scans.
Comparison of Coronary CT Angiography Image Quality With and Without Breast Shields.
                                  Angina.com Interview with Dr. Eddie Hulten, MD MPH
                                          Non-Invasive Cardiovascular Imaging
                                             Brigham and Women’s Hospital
                                                 Harvard Medical School
                                                         (cont)


•   Angina.com: What should clinicians and patients take away from this study?
•   Dr. Hulten: Patients scanned with breast shields in this study had interpretable coronary CT
    angiograms although the scans were noisier and had different plaque types detected, which could
    be associated with the shielding. However, breast shields are not currently recommended for
    clinical use by the SCCT or medical physicists due to concern for wasting radiation. Also other
    radiation sparing techniques may be more effective, reproducible, and impact image noise less.

•   Angina.com: What further research do you recommend as a result of your study?
•   Dr. Hulten: Study of additional radiation reducing methods during coronary CT angiography is
    indicated. While breast shields are not recommended for routine clinical use, further evaluation of
    shields’ impact upon coronary CT imaging may be warranted in specific research protocols.

•   Citation:
•   Comparison of Coronary CT Angiography Image Quality With and Without Breast Shields.
•   Hulten E, Devine P, Welch T, Feuerstein I, Taylor A, Petrillo S, Luncheon M, Nguyen B, Villines TC.
•   Cardiology Service, Walter Reed National Military Medical Center, Bethesda, MD.
    AJR Am J Roentgenol. 2013 Mar;200(3):529-36. doi: 10.2214/AJR.11.8302.
Trajectories of cardiometabolic risk factors before diagnosis of three subtypes of type 2 diabetes
                               Angina.com Interview with: Dr Kristine Færch PhD
                               BSc, MSc (Human Nutrison), PhD. STAR Researcher
                                               Steno Diabetes Center
                                                Niels Steensens Vej 1
                                           DK-2820 Gentofte Denmark



• Angina.com: What are the main findings of the study?
• Dr. Færch: Our study underscores the heterogeneity of
  type 2 diabetes. Individuals who develop type 2
  diabetes have different underlying mechanisms
  dependent on whether they are diagnosed with
  diabetes by elevated fasting glucose, elevated 2-hour
  glucose (after an oral glucose tolerance test) or
  combined elevated fasting and 2-hour glucose. We
  found very different trajectories of beta cell function,
  obesity development and several cardiovascular risk
  factors prior to diagnosis of diabetes in the three
  groups.
Trajectories of cardiometabolic risk factors before diagnosis of three subtypes of type 2 diabetes
                                   Angina.com Interview with: Dr Kristine Færch PhD
                                   BSc, MSc (Human Nutrison), PhD. STAR Researcher
                                                   Steno Diabetes Center
                                                    Niels Steensens Vej 1
                                               DK-2820 Gentofte Denmark
                                                            (cont)


• Angina.com: Were any of the findings unexpected?
• Dr. Færch: We expected to find different patterns of some cardiometabolic
  risk factors, but we were surprised that the differences were so
  pronounced in the different subtypes of type 2 diabetes – especially with
  regard to beta cell function. A classic pattern of early beta cell
  compensation followed by a progressive loss of beta cell function was only
  found in 25% of the individuals who developed diabetes, whereas 20%
  had a constant low beta cell function starting already 18 years before they
  were diagnosed with diabetes.
• Angina.com: What should clinicians and patients take away from this
  study?
• Dr. Færch: Most clinicians already acknowledge that type 2 diabetes is a
  heterogeneous disease, but this study adds to a deeper understanding of
  this heterogeneity. It is likely that the optimal treatment will differ among
  patients with the different subtypes identified in our study.
Trajectories of cardiometabolic risk factors before diagnosis of three subtypes of type 2 diabetes
                                    Angina.com Interview with: Dr Kristine Færch PhD
                                    BSc, MSc (Human Nutrison), PhD. STAR Researcher
                                                    Steno Diabetes Center
                                                     Niels Steensens Vej 1
                                                DK-2820 Gentofte Denmark
                                                             (cont)

•   Angina.com: What further research do you recommend as a result of your study?
•   Dr. Færch: First of all, it is important to study the underlying mechanisms of the
    different type 2 diabetes subtypes in other populations because there may be
    ethnic differences. If our findings are confirmed in other populations, the next step
    will be to see how the different subtypes of type 2 diabetes respond to medication
    in regard to glucose control and prevention of diabetic complications. Also, it will
    be relevant to examine whether type 2 diabetes should be prevented differently in
    high-risk individuals with impaired fasting glycemia (elevated fasting glucose) and
    impaired glucose tolerance (elevated 2-hour glucose).
•   Citation:
•   Trajectories of cardiometabolic risk factors before diagnosis of three subtypes of
    type 2 diabetes: a post-hoc analysis of the longitudinal Whitehall II cohort study
    Dr Kristine Færch PhD,Daniel R Witte PhD,Adam G Tabák PhD,Leigh Perreault
    MD,Christian Herder PhD,Eric J Brunner PhD,Prof Mika Kivimäki PhD,Dorte Vistisen
    PhD
    The Lancet Diabetes & Endocrinology – 21 February 2013
    DOI: 10.1016/S2213-8587(13)70008-1
African American Race is a Correlate of Heart Failure in Breast Cancer Survivors: A study of 26,347 women identified with breast cancer
                                                              from 1973-2007
                                         Angina.com Interview with Anna Valina-Toth, MD, PhD
                                                     Department of Internal Medicine
                                            Case Western Reserve University at MetroHealth



• Angina.com: What are the main findings of the study?
• Answer: – In female breast cancer survivors, the risk of developing heart
  failure is higher in African Americans compared to Caucasians.

• Angina.com: Were any of the findings unexpected?
• Answer: - We are not surprised that in breast cancer survivors, African
  Americans have higher heart failure occurrence compared to
  whites. African Americans, especially women, relative to whites, have
  excessive prevalence of hypertension, obesity and obesity related diseases
  (including diabetes), oxidative stress, inflammation, and endothelium
  nitric oxide-dependent vascular dysfunction and vitamin D deficiency.
• All these aforementioned co-morbidities plausibly predispose African
  Americans to higher risk of HF events including those who are breast
  cancer survivors.
African American Race is a Correlate of Heart Failure in Breast Cancer Survivors: A study of 26,347 women identified with breast cancer
                                                                 from 1973-2007
                                            Angina.com Interview with Anna Valina-Toth, MD, PhD
                                                        Department of Internal Medicine
                                               Case Western Reserve University at MetroHealth
                                                                      (cont)


•       Angina.com: What should clinicians and patients take away from this study?
•       Answer:- This study reveals a significant health disparity in heart failure prevalence
        that disproportionately affects African Americans compared to Caucasians in a
        cohort of breast cancer survivors.

•       Angina.com: What further research do you recommend as a result of your study?
•       Answer:- Future research to determine whether non-invasive cardiac imaging and
        pre-treatment with cardioprotective drugs such as ACE-inhibitors and beta
        blockers prior to initiation of antracyclines and trastuzumab based-chemotherapy
        would reduce the risk of heart failure in breast cancer patients especially in African
        Americans who are predisposed to developing heart failure merit investigation.
•       Citation:
•       African American Race is a Correlate of Heart Failure in Breast Cancer Survivors: A
        study of 26,347 women identified with breast cancer from 1973-2007 Presented at
        ACC 2013 March 2013
Tooth Loss Is Highly Prevalent And Associated With Cardiovascular Risk Factors In Patients With Chronic Coronary Heart Disease In The
                                                         Global Stability Trial
                                            Angina.com Interview with Dr. Ola Vedin, M.D.
                                 Department of Medical Sciences/Uppsala Clinical Research Centre (UCR)
                                                 Uppsala University Uppsala SWEDEN




• Angina.com: What are the main findings of the study?
• Dr. Vedin: Self-reported tooth loss and gum bleeding, as markers of
  periodontal disease, are common in this global population of
  patients with established coronary heart disease (i.e. previous
  myocardial infarction or verified coronary artery stenosis). Poor
  dental health, especially tooth loss, is associated with several
  established cardiovascular risk factors, including diabetes, smoking,
  high blood pressure and obesity, but also with more novel ones
  including Lp-PLA2.
  Angina.com: Were any of the findings unexpected?

• Dr. Vedin: We were surprised by the large proportion of patients
  with no or very few teeth and had expected somewhat stronger
  associations between gum bleeding and cardiovascular risk factors.
Tooth Loss Is Highly Prevalent And Associated With Cardiovascular Risk Factors In Patients With Chronic Coronary Heart Disease In The
                                                        Global Stability Trial
                                           Angina.com Interview with Dr. Ola Vedin, M.D.
                                Department of Medical Sciences/Uppsala Clinical Research Centre (UCR)
                                                Uppsala University Uppsala SWEDEN
                                                               (cont)



• Angina.com: What should clinicians and
  patients take away from this study?
• Dr. Vedin: There is an association between
  self-reported tooth loss and cardiovascular
  risk factors but we do not yet know whether
  this relationship is causal.
• Periodontal disease in relation to
  cardiovascular risk is a hot, but poorly
  understood topic that needs further exploring.
Tooth Loss Is Highly Prevalent And Associated With Cardiovascular Risk Factors In Patients With Chronic Coronary Heart Disease In The
                                                            Global Stability Trial
                                               Angina.com Interview with Dr. Ola Vedin, M.D.
                                    Department of Medical Sciences/Uppsala Clinical Research Centre (UCR)
                                                    Uppsala University Uppsala SWEDEN
                                                                   (cont)


•      Angina.com: What further research do you recommend as a result of your study?
•      Dr. Vedin: We are anticipating outcome data later this year when the study closes.
       Then we can relate the data on dental health to cardiovascular outcome and, after
       adjusting for common risk factors, clarify whether self-reported tooth loss and
       gum bleeding may actually be associated with myocardial infarction and death.
•      More widely speaking, there is a need for independent studies designed to
       determine whether poor dental health, or periodontal disease, actually causes
       coronary heart disease and if so, if periodontal treatment lowers cardiovascular
       risk.
•      Citation:
•      Tooth Loss Is Highly Prevalent And Associated With Cardiovascular Risk Factors In
       Patients With Chronic Coronary Heart Disease In The Global Stability Trial
•      Dr. Vedin presented the study “Tooth Loss Is Highly Prevalent And Associated With
       Cardiovascular Risk Factors In Patients With Chronic Coronary Heart Disease In The
       Global Stability Trial” on Saturday, March 9 at 3:45 p.m., in Moscone Center, Expo
       North during the ACC13 meeting
Sleep Apnea and Risk of Peptic Ulcer Bleeding
                Angina.com Interview with: Chou, Kun-Ta, M.D
                Taipei Veterans General Hospital, Taipei, Taiwan




• Angina.com: What are the main findings of the
  study?
• Response: In this retrospective matched-control
  cohort study including 35,480 patients (7,096
  sleep apnea patients vs. 28,384 controls) with a
  mean follow-up period of 3.57 years, we found
  patients with sleep apnea experienced a 2.400-
  fold (95% CI, 1.544-3.731, p<0.001) higher risk for
  incident peptic ulcer bleeding after adjusting
  other significant variables.
Sleep Apnea and Risk of Peptic Ulcer Bleeding
                  Angina.com Interview with: Chou, Kun-Ta, M.D
                  Taipei Veterans General Hospital, Taipei, Taiwan
                                      (cont)



• Angina.com: Were any of the findings unexpected?
• Response: Sleep apnea is a common disease and
  linked with a variety of cardiovascular diseases, such as
  hypertension, atherosclerosis, and arrhythmia.
  However, its impact on the affected individuals seems
  more widespread than we would expect, not
  exclusively limited to the cardiovascular system. The
  gastrointestinal system is probable to be affected as
  well. We found it surprising that sleep apnea confers a
  higher risk for incident peptic ulcer bleeding, even
  within a relatively short follow-up period in such a
  young population (aged 46.61 years on average).
Sleep Apnea and Risk of Peptic Ulcer Bleeding
                    Angina.com Interview with: Chou, Kun-Ta, M.D
                    Taipei Veterans General Hospital, Taipei, Taiwan
                                        (cont)



• Angina.com: What should clinicians and patients take
  away from this study?
• Response: Our results suggest that sleep apnea is an
  independent risk factor for peptic ulcer bleeding. Despite
  clear conclusion provided, this study is limited by its
  retrospective design and inability to include information
  about the lifestyle of the enrollees, such as smoking,
  alcohol consumption, obesity etc., which may possibly
  confound our results.
• For treatment of peptic ulcer bleeding in clinical practice,
  the clinicians should go through the traditional risk factors
  in such patients. For those without apparent risk factors
  identified, we may survey for sleep apnea as a potential
  predisposing factor.
Sleep Apnea and Risk of Peptic Ulcer Bleeding
                     Angina.com Interview with: Chou, Kun-Ta, M.D
                     Taipei Veterans General Hospital, Taipei, Taiwan
                                         (cont)



• Response: What further research do you recommend as a result
  of your study?
• Response: Our study may prompt further elaborate prospective
  research to confirm our conclusion and probe into the mechanism
  underlying the linkage of sleep apnea and its gastrointestinal
  complications.
• Citation:
• Sleep Apnea and Risk of Peptic Ulcer Bleeding: A Nationwide
  Population-based Study
• Am J Med. 2013 Mar;126(3):249-255.e1. doi:
  10.1016/j.amjmed.2012.08.017.
• Shiao TH, Liu CJ, Luo JC, Su KC, Chen YM, Chen TJ, Chou KT, Shiao
  GM, Lee YC.
  Department of Chest Medicine, Taipei Veterans General Hospital,
  Taipei, Taiwan.
The Effect of Weight Loss on the Incidence of Heart Failure or Death in MADIT-CRT Patients
                        Angina.com Interview with Dr. Valentina Kutyifa, MD, MSc
                                 University of Rochester Medical Center NY




• Angina.com: What are the main findings of the study?
• Dr. Kutyifa: In 170 of 994 (17%) patients implanted
  with CRT-D in the MADIT-CRT clinical trial, an
  unplanned, greater than – 2 kg weight loss was
  observed at 12 months after device implantation. The
  risk of heart failure or death was significantly higher in
  patients with unanticipated, significant weight loss as
  compared to those without weight loss. Each kilogram
  weight loss was associated with 4% increase in the risk
  of heart failure or death during an average of 29
  months follow-up in the study. Patients with left
  bundle branch block and significant weight loss (> –
  2kg) doubled their risk of heart failure or death.
The Effect of Weight Loss on the Incidence of Heart Failure or Death in MADIT-CRT Patients
                            Angina.com Interview with Dr. Valentina Kutyifa, MD, MSc
                                     University of Rochester Medical Center NY
                                                       (cont)



•   Angina.com: Were any of the findings unexpected?
•   Dr. Kutyifa: This study is consistent with the findings from other studies that have
    looked at the obesity paradox, suggesting that patients with weight loss have
    worse clinical outcome as compared to those without a weight loss. We need to
    stress that in this particular study, patients were not following any dietary rules,
    and they were not encouraged to lose weight, the weight loss was unanticipated.
•   Angina.com: What should clinicians and patients take away from this study?
•   Dr. Kutyifa: In patients with heart failure who receive a cardiac resynchronization
    device with defibrillator (CRT-D), unintended weight loss increases the risk of heart
    failure or death.
•   It may be advisable for heart failure patients with unplanned significant weight
    loss to consult their doctor, and physicians may perform diagnostic tests, refining
    medical treatment if necessary, and monitor these patients more closely to
    prevent the development of serious events.
•   However, we still suggest implementing weight control under a physician’s
    supervision in obese heart failure patients, because obesity is known to be
    associated with diabetes, high blood pressure, and the development of coronary
    artery disease.
The Effect of Weight Loss on the Incidence of Heart Failure or Death in MADIT-CRT Patients
                          Angina.com Interview with Dr. Valentina Kutyifa, MD, MSc
                                   University of Rochester Medical Center NY
                                                     (cont)



• Angina.com: What further research do you recommend as a result of
  your study?
• Dr. Kutyifa: The effects of weight loss needs to be further evaluated in
  patients with an implanted CRT-D, and baseline obesity needs to be taken
  into consideration. It would be interesting to see, if this effect is
  maintained regardless of the baseline weight, and baseline body mass
  index (BMI), or less obese patients would more affected than those with
  obesity to start with.
• Citation:
• The Effect of Weight Loss on the Incidence of Heart Failure or Death in
  MADIT-CRT Patients
• American College of Cardiology annual meeting in San Francisco
  Abstract: 308
  Valentina Kutyifa ,Mehmet Aktas, David Huang, Scott McNitt, Arthur Moss,
  Wojciech Zareba, University of Rochester Medical Center, Heart Research
  FollowUp Program, Rochester, NY, USA
Association of Body Mass Index with Risks of Myocardial Infarction and Ischemic Stroke in Young Women
                                 Angina.com Interview with Michelle Schmiegelow
                                   PhD student at the University of Copenhagen




• Angina.com: What are the main findings of the study?
• Answer: In this nationwide register-based study of 273,000 healthy
  women with a median age of approximately 30 years, we found
  obesity (body mass index >30 kg/m2) to be associated with a
  significantly increased risk of myocardial infarction and ischemic
  stroke within 4.5 years following childbirth. The absolute risks were
  low, but the key message is clear.

• Angina.com: Were any of the findings unexpected?
• Answer: We found it surprising that the obesity-associated health
  risk becomes apparent within a relatively short follow-up in such a
  young population of women even following adjustment for
  important cardiovascular risk factors.
Angina.com slideshare march 24 2013
Angina.com slideshare march 24 2013
Angina.com slideshare march 24 2013
Angina.com slideshare march 24 2013
Angina.com slideshare march 24 2013
Angina.com slideshare march 24 2013
Angina.com slideshare march 24 2013
Angina.com slideshare march 24 2013
Angina.com slideshare march 24 2013
Angina.com slideshare march 24 2013
Angina.com slideshare march 24 2013
Angina.com slideshare march 24 2013
Angina.com slideshare march 24 2013
Angina.com slideshare march 24 2013
Angina.com slideshare march 24 2013
Angina.com slideshare march 24 2013
Angina.com slideshare march 24 2013
Angina.com slideshare march 24 2013
Angina.com slideshare march 24 2013
Angina.com slideshare march 24 2013
Angina.com slideshare march 24 2013
Angina.com slideshare march 24 2013
Angina.com slideshare march 24 2013
Angina.com slideshare march 24 2013
Angina.com slideshare march 24 2013
Angina.com slideshare march 24 2013
Angina.com slideshare march 24 2013
Angina.com slideshare march 24 2013
Angina.com slideshare march 24 2013
Angina.com slideshare march 24 2013
Angina.com slideshare march 24 2013
Angina.com slideshare march 24 2013
Angina.com slideshare march 24 2013
Angina.com slideshare march 24 2013
Angina.com slideshare march 24 2013
Angina.com slideshare march 24 2013

More Related Content

What's hot

Should I Participate in a Clinical Trial?
Should I Participate in a Clinical Trial?Should I Participate in a Clinical Trial?
Should I Participate in a Clinical Trial?Summit Health
 
Strategies to improve adherence to antihypertensive medication
Strategies to improve adherence to antihypertensive medicationStrategies to improve adherence to antihypertensive medication
Strategies to improve adherence to antihypertensive medicationmagdy elmasry
 
Clinical thyroidology guidelines
Clinical thyroidology guidelinesClinical thyroidology guidelines
Clinical thyroidology guidelinesPrasanna Datta
 
MedicalResearch.com: Medical Research Exclusive Interviews April 8 2015
MedicalResearch.com:  Medical Research Exclusive Interviews April 8  2015MedicalResearch.com:  Medical Research Exclusive Interviews April 8  2015
MedicalResearch.com: Medical Research Exclusive Interviews April 8 2015Marie Benz MD FAAD
 
MedicalResearch.com: Medical Research Exclusive Interviews December 4 2014
MedicalResearch.com:  Medical Research Exclusive Interviews December 4  2014MedicalResearch.com:  Medical Research Exclusive Interviews December 4  2014
MedicalResearch.com: Medical Research Exclusive Interviews December 4 2014Marie Benz MD FAAD
 
Maths statistics report
Maths statistics reportMaths statistics report
Maths statistics reportashleyyeap
 
MedicalResearch.com: Medical Research Exclusive Interviews November 21 2014
MedicalResearch.com:  Medical Research Exclusive Interviews November 21 2014MedicalResearch.com:  Medical Research Exclusive Interviews November 21 2014
MedicalResearch.com: Medical Research Exclusive Interviews November 21 2014Marie Benz MD FAAD
 
Lifestyle predictors in healthy aging men
Lifestyle predictors in healthy aging menLifestyle predictors in healthy aging men
Lifestyle predictors in healthy aging menMarc Evans Abat
 
MedicalResearch.com: Medical Research Exclusive Interviews March 17 2015
MedicalResearch.com:  Medical Research Exclusive Interviews March 17 2015MedicalResearch.com:  Medical Research Exclusive Interviews March 17 2015
MedicalResearch.com: Medical Research Exclusive Interviews March 17 2015Marie Benz MD FAAD
 
MedicalResearch.com: Medical Research Exclusive Interviews May 12 2015
MedicalResearch.com:  Medical Research Exclusive Interviews May 12 2015MedicalResearch.com:  Medical Research Exclusive Interviews May 12 2015
MedicalResearch.com: Medical Research Exclusive Interviews May 12 2015Marie Benz MD FAAD
 
[ppt] RCpsych - Failing medical care of psychiatric patients (vMar11)
[ppt] RCpsych - Failing medical care of psychiatric patients (vMar11)[ppt] RCpsych - Failing medical care of psychiatric patients (vMar11)
[ppt] RCpsych - Failing medical care of psychiatric patients (vMar11)Alex J Mitchell
 
A Pre Experimental Study to Assess the Effectiveness of Structured Teaching P...
A Pre Experimental Study to Assess the Effectiveness of Structured Teaching P...A Pre Experimental Study to Assess the Effectiveness of Structured Teaching P...
A Pre Experimental Study to Assess the Effectiveness of Structured Teaching P...ijtsrd
 
Am 10.40 gardner
Am 10.40 gardnerAm 10.40 gardner
Am 10.40 gardnerplmiami
 
Abat wellness in elderly--pims 2020 version 2 -trimmed down
Abat wellness in elderly--pims 2020 version 2 -trimmed downAbat wellness in elderly--pims 2020 version 2 -trimmed down
Abat wellness in elderly--pims 2020 version 2 -trimmed downMarc Evans Abat
 
Public Health Screening: Programmes and Policy Lecture
Public Health Screening: Programmes and Policy LecturePublic Health Screening: Programmes and Policy Lecture
Public Health Screening: Programmes and Policy LectureDr Gary Kerr
 
Hemodialysis Nephrology Interviews March 9 2013
Hemodialysis Nephrology Interviews March 9 2013Hemodialysis Nephrology Interviews March 9 2013
Hemodialysis Nephrology Interviews March 9 2013Marie Benz
 
Oral Contraceptive Pills Use and Hypertension
Oral Contraceptive Pills Use and HypertensionOral Contraceptive Pills Use and Hypertension
Oral Contraceptive Pills Use and Hypertensioninventionjournals
 

What's hot (20)

Should I Participate in a Clinical Trial?
Should I Participate in a Clinical Trial?Should I Participate in a Clinical Trial?
Should I Participate in a Clinical Trial?
 
Strategies to improve adherence to antihypertensive medication
Strategies to improve adherence to antihypertensive medicationStrategies to improve adherence to antihypertensive medication
Strategies to improve adherence to antihypertensive medication
 
Clinical thyroidology guidelines
Clinical thyroidology guidelinesClinical thyroidology guidelines
Clinical thyroidology guidelines
 
MedicalResearch.com: Medical Research Exclusive Interviews April 8 2015
MedicalResearch.com:  Medical Research Exclusive Interviews April 8  2015MedicalResearch.com:  Medical Research Exclusive Interviews April 8  2015
MedicalResearch.com: Medical Research Exclusive Interviews April 8 2015
 
MedicalResearch.com: Medical Research Exclusive Interviews December 4 2014
MedicalResearch.com:  Medical Research Exclusive Interviews December 4  2014MedicalResearch.com:  Medical Research Exclusive Interviews December 4  2014
MedicalResearch.com: Medical Research Exclusive Interviews December 4 2014
 
Maths statistics report
Maths statistics reportMaths statistics report
Maths statistics report
 
MedicalResearch.com: Medical Research Exclusive Interviews November 21 2014
MedicalResearch.com:  Medical Research Exclusive Interviews November 21 2014MedicalResearch.com:  Medical Research Exclusive Interviews November 21 2014
MedicalResearch.com: Medical Research Exclusive Interviews November 21 2014
 
Lifestyle predictors in healthy aging men
Lifestyle predictors in healthy aging menLifestyle predictors in healthy aging men
Lifestyle predictors in healthy aging men
 
MedicalResearch.com: Medical Research Exclusive Interviews March 17 2015
MedicalResearch.com:  Medical Research Exclusive Interviews March 17 2015MedicalResearch.com:  Medical Research Exclusive Interviews March 17 2015
MedicalResearch.com: Medical Research Exclusive Interviews March 17 2015
 
MedicalResearch.com: Medical Research Exclusive Interviews May 12 2015
MedicalResearch.com:  Medical Research Exclusive Interviews May 12 2015MedicalResearch.com:  Medical Research Exclusive Interviews May 12 2015
MedicalResearch.com: Medical Research Exclusive Interviews May 12 2015
 
[ppt] RCpsych - Failing medical care of psychiatric patients (vMar11)
[ppt] RCpsych - Failing medical care of psychiatric patients (vMar11)[ppt] RCpsych - Failing medical care of psychiatric patients (vMar11)
[ppt] RCpsych - Failing medical care of psychiatric patients (vMar11)
 
A Pre Experimental Study to Assess the Effectiveness of Structured Teaching P...
A Pre Experimental Study to Assess the Effectiveness of Structured Teaching P...A Pre Experimental Study to Assess the Effectiveness of Structured Teaching P...
A Pre Experimental Study to Assess the Effectiveness of Structured Teaching P...
 
Am 10.40 gardner
Am 10.40 gardnerAm 10.40 gardner
Am 10.40 gardner
 
Regression of Coronary Atherosclerosis through Healthy Lifestyle in Coronary ...
Regression of Coronary Atherosclerosis through Healthy Lifestyle in Coronary ...Regression of Coronary Atherosclerosis through Healthy Lifestyle in Coronary ...
Regression of Coronary Atherosclerosis through Healthy Lifestyle in Coronary ...
 
journal club- dual-antiplatelets therapy Post AMI
journal club- dual-antiplatelets therapy Post AMIjournal club- dual-antiplatelets therapy Post AMI
journal club- dual-antiplatelets therapy Post AMI
 
Abat wellness in elderly--pims 2020 version 2 -trimmed down
Abat wellness in elderly--pims 2020 version 2 -trimmed downAbat wellness in elderly--pims 2020 version 2 -trimmed down
Abat wellness in elderly--pims 2020 version 2 -trimmed down
 
Public Health Screening: Programmes and Policy Lecture
Public Health Screening: Programmes and Policy LecturePublic Health Screening: Programmes and Policy Lecture
Public Health Screening: Programmes and Policy Lecture
 
Hemodialysis Nephrology Interviews March 9 2013
Hemodialysis Nephrology Interviews March 9 2013Hemodialysis Nephrology Interviews March 9 2013
Hemodialysis Nephrology Interviews March 9 2013
 
Oral Contraceptive Pills Use and Hypertension
Oral Contraceptive Pills Use and HypertensionOral Contraceptive Pills Use and Hypertension
Oral Contraceptive Pills Use and Hypertension
 
Daily Health Update 05-04-17 Rode Chiropractic
Daily Health Update 05-04-17 Rode ChiropracticDaily Health Update 05-04-17 Rode Chiropractic
Daily Health Update 05-04-17 Rode Chiropractic
 

Viewers also liked

Hemodialysis.com | Kidney Disease | ESRD | Dialysis
Hemodialysis.com | Kidney Disease | ESRD | DialysisHemodialysis.com | Kidney Disease | ESRD | Dialysis
Hemodialysis.com | Kidney Disease | ESRD | DialysisMarie Benz MD FAAD
 
MedicalResearch.com: Medical Research Exclusive Interviews December 14 2014
MedicalResearch.com:  Medical Research Exclusive Interviews December 14 2014MedicalResearch.com:  Medical Research Exclusive Interviews December 14 2014
MedicalResearch.com: Medical Research Exclusive Interviews December 14 2014Marie Benz MD FAAD
 
Hemodialysis.com | Hemodialysis | Dialysis | Kidney Disease
Hemodialysis.com | Hemodialysis | Dialysis | Kidney DiseaseHemodialysis.com | Hemodialysis | Dialysis | Kidney Disease
Hemodialysis.com | Hemodialysis | Dialysis | Kidney DiseaseMarie Benz MD FAAD
 
renal replacement therapies
renal replacement therapiesrenal replacement therapies
renal replacement therapiesRia Saira
 
Nickel Titanium Instruments in Endodontics: Part 2
Nickel Titanium Instruments in Endodontics: Part 2Nickel Titanium Instruments in Endodontics: Part 2
Nickel Titanium Instruments in Endodontics: Part 2Ashok Ayer
 
CLEANING AND SHAPING USING ROTARY ENDODONTIC INSTRUMENTS /certified fixed or...
CLEANING AND SHAPING USING ROTARY ENDODONTIC INSTRUMENTS  /certified fixed or...CLEANING AND SHAPING USING ROTARY ENDODONTIC INSTRUMENTS  /certified fixed or...
CLEANING AND SHAPING USING ROTARY ENDODONTIC INSTRUMENTS /certified fixed or...Indian dental academy
 
Root canal anatomy and access cavities
Root canal anatomy and access cavitiesRoot canal anatomy and access cavities
Root canal anatomy and access cavitiesmilanchande
 
Root Canal Treatment
Root Canal TreatmentRoot Canal Treatment
Root Canal Treatmentendodontics
 
Access cavity preparation
Access cavity preparationAccess cavity preparation
Access cavity preparationAnkita Varshney
 
textbook of Oral medicine (ingles)
 textbook of  Oral medicine (ingles) textbook of  Oral medicine (ingles)
textbook of Oral medicine (ingles)Alex Avendaño
 
Mozcon 2016 - Building Links in 2016
Mozcon 2016 - Building Links in 2016Mozcon 2016 - Building Links in 2016
Mozcon 2016 - Building Links in 2016Kirsty Hulse
 
Cleaning and shaping the root canal system
Cleaning and shaping the root canal systemCleaning and shaping the root canal system
Cleaning and shaping the root canal systemParth Thakkar
 
SEO: Getting Personal
SEO: Getting PersonalSEO: Getting Personal
SEO: Getting PersonalKirsty Hulse
 

Viewers also liked (14)

Hemodialysis.com | Kidney Disease | ESRD | Dialysis
Hemodialysis.com | Kidney Disease | ESRD | DialysisHemodialysis.com | Kidney Disease | ESRD | Dialysis
Hemodialysis.com | Kidney Disease | ESRD | Dialysis
 
Kidney failure
Kidney failureKidney failure
Kidney failure
 
MedicalResearch.com: Medical Research Exclusive Interviews December 14 2014
MedicalResearch.com:  Medical Research Exclusive Interviews December 14 2014MedicalResearch.com:  Medical Research Exclusive Interviews December 14 2014
MedicalResearch.com: Medical Research Exclusive Interviews December 14 2014
 
Hemodialysis.com | Hemodialysis | Dialysis | Kidney Disease
Hemodialysis.com | Hemodialysis | Dialysis | Kidney DiseaseHemodialysis.com | Hemodialysis | Dialysis | Kidney Disease
Hemodialysis.com | Hemodialysis | Dialysis | Kidney Disease
 
renal replacement therapies
renal replacement therapiesrenal replacement therapies
renal replacement therapies
 
Nickel Titanium Instruments in Endodontics: Part 2
Nickel Titanium Instruments in Endodontics: Part 2Nickel Titanium Instruments in Endodontics: Part 2
Nickel Titanium Instruments in Endodontics: Part 2
 
CLEANING AND SHAPING USING ROTARY ENDODONTIC INSTRUMENTS /certified fixed or...
CLEANING AND SHAPING USING ROTARY ENDODONTIC INSTRUMENTS  /certified fixed or...CLEANING AND SHAPING USING ROTARY ENDODONTIC INSTRUMENTS  /certified fixed or...
CLEANING AND SHAPING USING ROTARY ENDODONTIC INSTRUMENTS /certified fixed or...
 
Root canal anatomy and access cavities
Root canal anatomy and access cavitiesRoot canal anatomy and access cavities
Root canal anatomy and access cavities
 
Root Canal Treatment
Root Canal TreatmentRoot Canal Treatment
Root Canal Treatment
 
Access cavity preparation
Access cavity preparationAccess cavity preparation
Access cavity preparation
 
textbook of Oral medicine (ingles)
 textbook of  Oral medicine (ingles) textbook of  Oral medicine (ingles)
textbook of Oral medicine (ingles)
 
Mozcon 2016 - Building Links in 2016
Mozcon 2016 - Building Links in 2016Mozcon 2016 - Building Links in 2016
Mozcon 2016 - Building Links in 2016
 
Cleaning and shaping the root canal system
Cleaning and shaping the root canal systemCleaning and shaping the root canal system
Cleaning and shaping the root canal system
 
SEO: Getting Personal
SEO: Getting PersonalSEO: Getting Personal
SEO: Getting Personal
 

Similar to Angina.com slideshare march 24 2013

Medical research slideshare_june_18_2015
Medical research slideshare_june_18_2015 Medical research slideshare_june_18_2015
Medical research slideshare_june_18_2015 Marie Benz MD FAAD
 
Medical research slideshare_june_18_2015
Medical research slideshare_june_18_2015 Medical research slideshare_june_18_2015
Medical research slideshare_june_18_2015 Marie Benz MD FAAD
 
MedicalResearch.com: Medical Research Exclusive Interviews July 9 2015
MedicalResearch.com:  Medical Research Exclusive Interviews July 9 2015MedicalResearch.com:  Medical Research Exclusive Interviews July 9 2015
MedicalResearch.com: Medical Research Exclusive Interviews July 9 2015Marie Benz MD FAAD
 
MedicalResearch.com: Medical Research Interviews September 22 2014
MedicalResearch.com:  Medical Research Interviews September 22 2014MedicalResearch.com:  Medical Research Interviews September 22 2014
MedicalResearch.com: Medical Research Interviews September 22 2014Marie Benz MD FAAD
 
MedicalResearch.com - Medical Research Interviews Week in Review
MedicalResearch.com - Medical Research Interviews Week in ReviewMedicalResearch.com - Medical Research Interviews Week in Review
MedicalResearch.com - Medical Research Interviews Week in ReviewMarie Benz MD FAAD
 
MedicalResearch.com: Medical Research Exclusive Interviews May 26 2015
MedicalResearch.com:  Medical Research Exclusive Interviews May 26 2015MedicalResearch.com:  Medical Research Exclusive Interviews May 26 2015
MedicalResearch.com: Medical Research Exclusive Interviews May 26 2015Marie Benz MD FAAD
 
MedicalResearch.com: Medical Research Interviews Month in Review
MedicalResearch.com:  Medical Research Interviews Month in ReviewMedicalResearch.com:  Medical Research Interviews Month in Review
MedicalResearch.com: Medical Research Interviews Month in ReviewMarie Benz MD FAAD
 
MedicalResearch.com: Medical Research Interviews September 3 2014
MedicalResearch.com:  Medical Research Interviews September 3 2014MedicalResearch.com:  Medical Research Interviews September 3 2014
MedicalResearch.com: Medical Research Interviews September 3 2014Marie Benz MD FAAD
 
MedicalResearch.com - Medical Research Week in Review
MedicalResearch.com - Medical Research  Week in ReviewMedicalResearch.com - Medical Research  Week in Review
MedicalResearch.com - Medical Research Week in ReviewMarie Benz MD FAAD
 
MedicalResearch.com - Medical Research Interviews Week in Review
MedicalResearch.com - Medical Research Interviews Week in ReviewMedicalResearch.com - Medical Research Interviews Week in Review
MedicalResearch.com - Medical Research Interviews Week in ReviewMarie Benz MD FAAD
 
From Overtreatment to Patient-Centered Care
From Overtreatment to  Patient-Centered CareFrom Overtreatment to  Patient-Centered Care
From Overtreatment to Patient-Centered CareNASHP HealthPolicy
 
MedicalResearch.com: Medical Research Exclusive Interviews April 1 2015
MedicalResearch.com:  Medical Research Exclusive Interviews April 1  2015MedicalResearch.com:  Medical Research Exclusive Interviews April 1  2015
MedicalResearch.com: Medical Research Exclusive Interviews April 1 2015Marie Benz MD FAAD
 
MedicalResearch.com: Medical Research Exclusive Interviews February 10 2015
MedicalResearch.com:  Medical Research Exclusive Interviews February 10  2015MedicalResearch.com:  Medical Research Exclusive Interviews February 10  2015
MedicalResearch.com: Medical Research Exclusive Interviews February 10 2015Marie Benz MD FAAD
 
MedicalResearch.com: Medical Research Interviews
MedicalResearch.com:  Medical Research InterviewsMedicalResearch.com:  Medical Research Interviews
MedicalResearch.com: Medical Research InterviewsMarie Benz MD FAAD
 
MedicalResearch.com: Medical Research Exclusive Interviews January 28 2015
MedicalResearch.com:  Medical Research Exclusive Interviews January 28 2015MedicalResearch.com:  Medical Research Exclusive Interviews January 28 2015
MedicalResearch.com: Medical Research Exclusive Interviews January 28 2015Marie Benz MD FAAD
 
Medical research slideshare_feb-24_2015
Medical research slideshare_feb-24_2015Medical research slideshare_feb-24_2015
Medical research slideshare_feb-24_2015Marie Benz MD FAAD
 

Similar to Angina.com slideshare march 24 2013 (20)

Medical research slideshare_june_18_2015
Medical research slideshare_june_18_2015 Medical research slideshare_june_18_2015
Medical research slideshare_june_18_2015
 
Medical research slideshare_june_18_2015
Medical research slideshare_june_18_2015 Medical research slideshare_june_18_2015
Medical research slideshare_june_18_2015
 
MedicalResearch.com: Medical Research Exclusive Interviews July 9 2015
MedicalResearch.com:  Medical Research Exclusive Interviews July 9 2015MedicalResearch.com:  Medical Research Exclusive Interviews July 9 2015
MedicalResearch.com: Medical Research Exclusive Interviews July 9 2015
 
MedicalResearch.com: Medical Research Interviews September 22 2014
MedicalResearch.com:  Medical Research Interviews September 22 2014MedicalResearch.com:  Medical Research Interviews September 22 2014
MedicalResearch.com: Medical Research Interviews September 22 2014
 
MedicalResearch.com - Medical Research Interviews Week in Review
MedicalResearch.com - Medical Research Interviews Week in ReviewMedicalResearch.com - Medical Research Interviews Week in Review
MedicalResearch.com - Medical Research Interviews Week in Review
 
MedicalResearch.com: Medical Research Exclusive Interviews May 26 2015
MedicalResearch.com:  Medical Research Exclusive Interviews May 26 2015MedicalResearch.com:  Medical Research Exclusive Interviews May 26 2015
MedicalResearch.com: Medical Research Exclusive Interviews May 26 2015
 
MedicalResearch.com: Medical Research Interviews Month in Review
MedicalResearch.com:  Medical Research Interviews Month in ReviewMedicalResearch.com:  Medical Research Interviews Month in Review
MedicalResearch.com: Medical Research Interviews Month in Review
 
MedicalResearch.com: Medical Research Interviews September 3 2014
MedicalResearch.com:  Medical Research Interviews September 3 2014MedicalResearch.com:  Medical Research Interviews September 3 2014
MedicalResearch.com: Medical Research Interviews September 3 2014
 
MedicalResearch.com - Medical Research Week in Review
MedicalResearch.com - Medical Research  Week in ReviewMedicalResearch.com - Medical Research  Week in Review
MedicalResearch.com - Medical Research Week in Review
 
Daily Health Update 05-15-17 Rode Chiropractic Poway CA
Daily Health Update 05-15-17  Rode Chiropractic Poway CADaily Health Update 05-15-17  Rode Chiropractic Poway CA
Daily Health Update 05-15-17 Rode Chiropractic Poway CA
 
Daily Health Update for 06/11/2015 from Poway Chiropractor Dr. Rode of Rode ...
Daily Health Update for  06/11/2015 from Poway Chiropractor Dr. Rode of Rode ...Daily Health Update for  06/11/2015 from Poway Chiropractor Dr. Rode of Rode ...
Daily Health Update for 06/11/2015 from Poway Chiropractor Dr. Rode of Rode ...
 
Daily Health Update for 3-24-15 for Rode Chiropractic in Poway CA
Daily Health Update for 3-24-15 for Rode Chiropractic in Poway CADaily Health Update for 3-24-15 for Rode Chiropractic in Poway CA
Daily Health Update for 3-24-15 for Rode Chiropractic in Poway CA
 
Daily Health Update 3-25-15 Dr. Rode of Rode Chiropractic in Poway CA 92064
Daily Health Update 3-25-15  Dr. Rode of Rode Chiropractic in Poway CA 92064Daily Health Update 3-25-15  Dr. Rode of Rode Chiropractic in Poway CA 92064
Daily Health Update 3-25-15 Dr. Rode of Rode Chiropractic in Poway CA 92064
 
MedicalResearch.com - Medical Research Interviews Week in Review
MedicalResearch.com - Medical Research Interviews Week in ReviewMedicalResearch.com - Medical Research Interviews Week in Review
MedicalResearch.com - Medical Research Interviews Week in Review
 
From Overtreatment to Patient-Centered Care
From Overtreatment to  Patient-Centered CareFrom Overtreatment to  Patient-Centered Care
From Overtreatment to Patient-Centered Care
 
MedicalResearch.com: Medical Research Exclusive Interviews April 1 2015
MedicalResearch.com:  Medical Research Exclusive Interviews April 1  2015MedicalResearch.com:  Medical Research Exclusive Interviews April 1  2015
MedicalResearch.com: Medical Research Exclusive Interviews April 1 2015
 
MedicalResearch.com: Medical Research Exclusive Interviews February 10 2015
MedicalResearch.com:  Medical Research Exclusive Interviews February 10  2015MedicalResearch.com:  Medical Research Exclusive Interviews February 10  2015
MedicalResearch.com: Medical Research Exclusive Interviews February 10 2015
 
MedicalResearch.com: Medical Research Interviews
MedicalResearch.com:  Medical Research InterviewsMedicalResearch.com:  Medical Research Interviews
MedicalResearch.com: Medical Research Interviews
 
MedicalResearch.com: Medical Research Exclusive Interviews January 28 2015
MedicalResearch.com:  Medical Research Exclusive Interviews January 28 2015MedicalResearch.com:  Medical Research Exclusive Interviews January 28 2015
MedicalResearch.com: Medical Research Exclusive Interviews January 28 2015
 
Medical research slideshare_feb-24_2015
Medical research slideshare_feb-24_2015Medical research slideshare_feb-24_2015
Medical research slideshare_feb-24_2015
 

More from Marie Benz MD FAAD

MedicalResearch.com: Medical Research Exclusive Interviews July 20 2015
MedicalResearch.com:  Medical Research Exclusive Interviews July 20  2015MedicalResearch.com:  Medical Research Exclusive Interviews July 20  2015
MedicalResearch.com: Medical Research Exclusive Interviews July 20 2015Marie Benz MD FAAD
 
MedicalResearch.com: Medical Research Exclusive Interviews July 16 2015
MedicalResearch.com:  Medical Research Exclusive Interviews July 16 2015MedicalResearch.com:  Medical Research Exclusive Interviews July 16 2015
MedicalResearch.com: Medical Research Exclusive Interviews July 16 2015Marie Benz MD FAAD
 
MedicalResearch.com: Medical Research Exclusive Interviews July 2 2015
MedicalResearch.com:  Medical Research Exclusive Interviews July 2 2015MedicalResearch.com:  Medical Research Exclusive Interviews July 2 2015
MedicalResearch.com: Medical Research Exclusive Interviews July 2 2015Marie Benz MD FAAD
 
MedicalResearch.com: Medical Research Exclusive Interviews June 26 2015
MedicalResearch.com:  Medical Research Exclusive Interviews June 26 2015MedicalResearch.com:  Medical Research Exclusive Interviews June 26 2015
MedicalResearch.com: Medical Research Exclusive Interviews June 26 2015Marie Benz MD FAAD
 
Medical research slideshare_june_24_2015
Medical research slideshare_june_24_2015 Medical research slideshare_june_24_2015
Medical research slideshare_june_24_2015 Marie Benz MD FAAD
 
MedicalResearch.com: Medical Research Exclusive Interviews June 11 2015
MedicalResearch.com:  Medical Research Exclusive Interviews June 11 2015MedicalResearch.com:  Medical Research Exclusive Interviews June 11 2015
MedicalResearch.com: Medical Research Exclusive Interviews June 11 2015Marie Benz MD FAAD
 
MedicalResearch.com: Medical Research Exclusive Interviews June 6 2015
MedicalResearch.com:  Medical Research Exclusive Interviews June 6 2015MedicalResearch.com:  Medical Research Exclusive Interviews June 6 2015
MedicalResearch.com: Medical Research Exclusive Interviews June 6 2015Marie Benz MD FAAD
 
MedicalResearch.com: Medical Research Exclusive Interviews May 18 2015
MedicalResearch.com:  Medical Research Exclusive Interviews May 18 2015MedicalResearch.com:  Medical Research Exclusive Interviews May 18 2015
MedicalResearch.com: Medical Research Exclusive Interviews May 18 2015Marie Benz MD FAAD
 
MedicalResearch.com: Medical Research Exclusive Interviews April 28 2015
MedicalResearch.com:  Medical Research Exclusive Interviews April 28  2015MedicalResearch.com:  Medical Research Exclusive Interviews April 28  2015
MedicalResearch.com: Medical Research Exclusive Interviews April 28 2015Marie Benz MD FAAD
 
MedicalResearch.com: Medical Research Exclusive Interviews April 20 2015
MedicalResearch.com:  Medical Research Exclusive Interviews April 20  2015MedicalResearch.com:  Medical Research Exclusive Interviews April 20  2015
MedicalResearch.com: Medical Research Exclusive Interviews April 20 2015Marie Benz MD FAAD
 
MedicalResearch.com: Medical Research Exclusive Interviews March 24 2015
MedicalResearch.com:  Medical Research Exclusive Interviews March 24 2015MedicalResearch.com:  Medical Research Exclusive Interviews March 24 2015
MedicalResearch.com: Medical Research Exclusive Interviews March 24 2015Marie Benz MD FAAD
 
MedicalResearch.com: Medical Research Exclusive Interviews March 12 2015
MedicalResearch.com:  Medical Research Exclusive Interviews March 12 2015MedicalResearch.com:  Medical Research Exclusive Interviews March 12 2015
MedicalResearch.com: Medical Research Exclusive Interviews March 12 2015Marie Benz MD FAAD
 
Medical research slideshare_march 12_2015
Medical research slideshare_march 12_2015 Medical research slideshare_march 12_2015
Medical research slideshare_march 12_2015 Marie Benz MD FAAD
 
MedicalResearch.com: Medical Research Exclusive Interviews March 5 2015
MedicalResearch.com:  Medical Research Exclusive Interviews March 5  2015MedicalResearch.com:  Medical Research Exclusive Interviews March 5  2015
MedicalResearch.com: Medical Research Exclusive Interviews March 5 2015Marie Benz MD FAAD
 
MedicalResearch.com: Medical Research Exclusive Interviews February 17 2015
MedicalResearch.com:  Medical Research Exclusive Interviews February 17  2015MedicalResearch.com:  Medical Research Exclusive Interviews February 17  2015
MedicalResearch.com: Medical Research Exclusive Interviews February 17 2015Marie Benz MD FAAD
 

More from Marie Benz MD FAAD (15)

MedicalResearch.com: Medical Research Exclusive Interviews July 20 2015
MedicalResearch.com:  Medical Research Exclusive Interviews July 20  2015MedicalResearch.com:  Medical Research Exclusive Interviews July 20  2015
MedicalResearch.com: Medical Research Exclusive Interviews July 20 2015
 
MedicalResearch.com: Medical Research Exclusive Interviews July 16 2015
MedicalResearch.com:  Medical Research Exclusive Interviews July 16 2015MedicalResearch.com:  Medical Research Exclusive Interviews July 16 2015
MedicalResearch.com: Medical Research Exclusive Interviews July 16 2015
 
MedicalResearch.com: Medical Research Exclusive Interviews July 2 2015
MedicalResearch.com:  Medical Research Exclusive Interviews July 2 2015MedicalResearch.com:  Medical Research Exclusive Interviews July 2 2015
MedicalResearch.com: Medical Research Exclusive Interviews July 2 2015
 
MedicalResearch.com: Medical Research Exclusive Interviews June 26 2015
MedicalResearch.com:  Medical Research Exclusive Interviews June 26 2015MedicalResearch.com:  Medical Research Exclusive Interviews June 26 2015
MedicalResearch.com: Medical Research Exclusive Interviews June 26 2015
 
Medical research slideshare_june_24_2015
Medical research slideshare_june_24_2015 Medical research slideshare_june_24_2015
Medical research slideshare_june_24_2015
 
MedicalResearch.com: Medical Research Exclusive Interviews June 11 2015
MedicalResearch.com:  Medical Research Exclusive Interviews June 11 2015MedicalResearch.com:  Medical Research Exclusive Interviews June 11 2015
MedicalResearch.com: Medical Research Exclusive Interviews June 11 2015
 
MedicalResearch.com: Medical Research Exclusive Interviews June 6 2015
MedicalResearch.com:  Medical Research Exclusive Interviews June 6 2015MedicalResearch.com:  Medical Research Exclusive Interviews June 6 2015
MedicalResearch.com: Medical Research Exclusive Interviews June 6 2015
 
MedicalResearch.com: Medical Research Exclusive Interviews May 18 2015
MedicalResearch.com:  Medical Research Exclusive Interviews May 18 2015MedicalResearch.com:  Medical Research Exclusive Interviews May 18 2015
MedicalResearch.com: Medical Research Exclusive Interviews May 18 2015
 
MedicalResearch.com: Medical Research Exclusive Interviews April 28 2015
MedicalResearch.com:  Medical Research Exclusive Interviews April 28  2015MedicalResearch.com:  Medical Research Exclusive Interviews April 28  2015
MedicalResearch.com: Medical Research Exclusive Interviews April 28 2015
 
MedicalResearch.com: Medical Research Exclusive Interviews April 20 2015
MedicalResearch.com:  Medical Research Exclusive Interviews April 20  2015MedicalResearch.com:  Medical Research Exclusive Interviews April 20  2015
MedicalResearch.com: Medical Research Exclusive Interviews April 20 2015
 
MedicalResearch.com: Medical Research Exclusive Interviews March 24 2015
MedicalResearch.com:  Medical Research Exclusive Interviews March 24 2015MedicalResearch.com:  Medical Research Exclusive Interviews March 24 2015
MedicalResearch.com: Medical Research Exclusive Interviews March 24 2015
 
MedicalResearch.com: Medical Research Exclusive Interviews March 12 2015
MedicalResearch.com:  Medical Research Exclusive Interviews March 12 2015MedicalResearch.com:  Medical Research Exclusive Interviews March 12 2015
MedicalResearch.com: Medical Research Exclusive Interviews March 12 2015
 
Medical research slideshare_march 12_2015
Medical research slideshare_march 12_2015 Medical research slideshare_march 12_2015
Medical research slideshare_march 12_2015
 
MedicalResearch.com: Medical Research Exclusive Interviews March 5 2015
MedicalResearch.com:  Medical Research Exclusive Interviews March 5  2015MedicalResearch.com:  Medical Research Exclusive Interviews March 5  2015
MedicalResearch.com: Medical Research Exclusive Interviews March 5 2015
 
MedicalResearch.com: Medical Research Exclusive Interviews February 17 2015
MedicalResearch.com:  Medical Research Exclusive Interviews February 17  2015MedicalResearch.com:  Medical Research Exclusive Interviews February 17  2015
MedicalResearch.com: Medical Research Exclusive Interviews February 17 2015
 

Recently uploaded

Different drug regularity bodies in different countries.
Different drug regularity bodies in different countries.Different drug regularity bodies in different countries.
Different drug regularity bodies in different countries.kishan singh tomar
 
Microbiology lecture presentation-1.pptx
Microbiology lecture presentation-1.pptxMicrobiology lecture presentation-1.pptx
Microbiology lecture presentation-1.pptxkitati1
 
Physiology of Smooth Muscles -Mechanics of contraction and relaxation
Physiology of Smooth Muscles -Mechanics of contraction and relaxationPhysiology of Smooth Muscles -Mechanics of contraction and relaxation
Physiology of Smooth Muscles -Mechanics of contraction and relaxationMedicoseAcademics
 
SGK RỐI LOẠN TOAN KIỀM ĐHYHN RẤT HAY VÀ ĐẶC SẮC.pdf
SGK RỐI LOẠN TOAN KIỀM ĐHYHN RẤT HAY VÀ ĐẶC SẮC.pdfSGK RỐI LOẠN TOAN KIỀM ĐHYHN RẤT HAY VÀ ĐẶC SẮC.pdf
SGK RỐI LOẠN TOAN KIỀM ĐHYHN RẤT HAY VÀ ĐẶC SẮC.pdfHongBiThi1
 
Role of Soap based and synthetic or syndets bar
Role of  Soap based and synthetic or syndets barRole of  Soap based and synthetic or syndets bar
Role of Soap based and synthetic or syndets barmohitRahangdale
 
EXERCISE PERFORMANCE.pptx, Lung function
EXERCISE PERFORMANCE.pptx, Lung functionEXERCISE PERFORMANCE.pptx, Lung function
EXERCISE PERFORMANCE.pptx, Lung functionkrishnareddy157915
 
QUESTIONS & ANSWERS FOR QUALITY ASSURANCE, RADIATIONBIOLOGY& RADIATION HAZARD...
QUESTIONS & ANSWERS FOR QUALITY ASSURANCE, RADIATIONBIOLOGY& RADIATION HAZARD...QUESTIONS & ANSWERS FOR QUALITY ASSURANCE, RADIATIONBIOLOGY& RADIATION HAZARD...
QUESTIONS & ANSWERS FOR QUALITY ASSURANCE, RADIATIONBIOLOGY& RADIATION HAZARD...Ganesan Yogananthem
 
Breast cancer -ONCO IN MEDICAL AND SURGICAL NURSING.pptx
Breast cancer -ONCO IN MEDICAL AND SURGICAL NURSING.pptxBreast cancer -ONCO IN MEDICAL AND SURGICAL NURSING.pptx
Breast cancer -ONCO IN MEDICAL AND SURGICAL NURSING.pptxNaveenkumar267201
 
ANATOMICAL FAETURES OF BONES FOR NURSING STUDENTS .pptx
ANATOMICAL FAETURES OF BONES  FOR NURSING STUDENTS .pptxANATOMICAL FAETURES OF BONES  FOR NURSING STUDENTS .pptx
ANATOMICAL FAETURES OF BONES FOR NURSING STUDENTS .pptxWINCY THIRUMURUGAN
 
Bulimia nervosa ( Eating Disorders) Mental Health Nursing.
Bulimia nervosa ( Eating Disorders) Mental Health Nursing.Bulimia nervosa ( Eating Disorders) Mental Health Nursing.
Bulimia nervosa ( Eating Disorders) Mental Health Nursing.aarjukhadka22
 
SGK RỐI LOẠN KALI MÁU CỰC KỲ QUAN TRỌNG.pdf
SGK RỐI LOẠN KALI MÁU CỰC KỲ QUAN TRỌNG.pdfSGK RỐI LOẠN KALI MÁU CỰC KỲ QUAN TRỌNG.pdf
SGK RỐI LOẠN KALI MÁU CỰC KỲ QUAN TRỌNG.pdfHongBiThi1
 
World-TB-Day-2023_Presentation_English.pptx
World-TB-Day-2023_Presentation_English.pptxWorld-TB-Day-2023_Presentation_English.pptx
World-TB-Day-2023_Presentation_English.pptxsumanchaulagain3
 
"Radical excision of DIE in subferile women with deep infiltrating endometrio...
"Radical excision of DIE in subferile women with deep infiltrating endometrio..."Radical excision of DIE in subferile women with deep infiltrating endometrio...
"Radical excision of DIE in subferile women with deep infiltrating endometrio...Sujoy Dasgupta
 
Basic structure of hair and hair growth cycle.pptx
Basic structure of hair and hair growth cycle.pptxBasic structure of hair and hair growth cycle.pptx
Basic structure of hair and hair growth cycle.pptxkomalt2001
 
ayurvedic formulations herbal drug technologyppt
ayurvedic formulations herbal drug technologypptayurvedic formulations herbal drug technologyppt
ayurvedic formulations herbal drug technologypptPradnya Wadekar
 
Trustworthiness of AI based predictions Aachen 2024
Trustworthiness of AI based predictions Aachen 2024Trustworthiness of AI based predictions Aachen 2024
Trustworthiness of AI based predictions Aachen 2024EwoutSteyerberg1
 
Red Blood Cells_anemia & polycythemia.pdf
Red Blood Cells_anemia & polycythemia.pdfRed Blood Cells_anemia & polycythemia.pdf
Red Blood Cells_anemia & polycythemia.pdfMedicoseAcademics
 
Physiotherapy Management of Rheumatoid Arthritis
Physiotherapy Management of Rheumatoid ArthritisPhysiotherapy Management of Rheumatoid Arthritis
Physiotherapy Management of Rheumatoid ArthritisNilofarRasheed1
 
The Importance of Mental Health: Why is Mental Health Important?
The Importance of Mental Health: Why is Mental Health Important?The Importance of Mental Health: Why is Mental Health Important?
The Importance of Mental Health: Why is Mental Health Important?Ryan Addison
 
How to cure cirrhosis and chronic hepatitis naturally
How to cure cirrhosis and chronic hepatitis naturallyHow to cure cirrhosis and chronic hepatitis naturally
How to cure cirrhosis and chronic hepatitis naturallyZurück zum Ursprung
 

Recently uploaded (20)

Different drug regularity bodies in different countries.
Different drug regularity bodies in different countries.Different drug regularity bodies in different countries.
Different drug regularity bodies in different countries.
 
Microbiology lecture presentation-1.pptx
Microbiology lecture presentation-1.pptxMicrobiology lecture presentation-1.pptx
Microbiology lecture presentation-1.pptx
 
Physiology of Smooth Muscles -Mechanics of contraction and relaxation
Physiology of Smooth Muscles -Mechanics of contraction and relaxationPhysiology of Smooth Muscles -Mechanics of contraction and relaxation
Physiology of Smooth Muscles -Mechanics of contraction and relaxation
 
SGK RỐI LOẠN TOAN KIỀM ĐHYHN RẤT HAY VÀ ĐẶC SẮC.pdf
SGK RỐI LOẠN TOAN KIỀM ĐHYHN RẤT HAY VÀ ĐẶC SẮC.pdfSGK RỐI LOẠN TOAN KIỀM ĐHYHN RẤT HAY VÀ ĐẶC SẮC.pdf
SGK RỐI LOẠN TOAN KIỀM ĐHYHN RẤT HAY VÀ ĐẶC SẮC.pdf
 
Role of Soap based and synthetic or syndets bar
Role of  Soap based and synthetic or syndets barRole of  Soap based and synthetic or syndets bar
Role of Soap based and synthetic or syndets bar
 
EXERCISE PERFORMANCE.pptx, Lung function
EXERCISE PERFORMANCE.pptx, Lung functionEXERCISE PERFORMANCE.pptx, Lung function
EXERCISE PERFORMANCE.pptx, Lung function
 
QUESTIONS & ANSWERS FOR QUALITY ASSURANCE, RADIATIONBIOLOGY& RADIATION HAZARD...
QUESTIONS & ANSWERS FOR QUALITY ASSURANCE, RADIATIONBIOLOGY& RADIATION HAZARD...QUESTIONS & ANSWERS FOR QUALITY ASSURANCE, RADIATIONBIOLOGY& RADIATION HAZARD...
QUESTIONS & ANSWERS FOR QUALITY ASSURANCE, RADIATIONBIOLOGY& RADIATION HAZARD...
 
Breast cancer -ONCO IN MEDICAL AND SURGICAL NURSING.pptx
Breast cancer -ONCO IN MEDICAL AND SURGICAL NURSING.pptxBreast cancer -ONCO IN MEDICAL AND SURGICAL NURSING.pptx
Breast cancer -ONCO IN MEDICAL AND SURGICAL NURSING.pptx
 
ANATOMICAL FAETURES OF BONES FOR NURSING STUDENTS .pptx
ANATOMICAL FAETURES OF BONES  FOR NURSING STUDENTS .pptxANATOMICAL FAETURES OF BONES  FOR NURSING STUDENTS .pptx
ANATOMICAL FAETURES OF BONES FOR NURSING STUDENTS .pptx
 
Bulimia nervosa ( Eating Disorders) Mental Health Nursing.
Bulimia nervosa ( Eating Disorders) Mental Health Nursing.Bulimia nervosa ( Eating Disorders) Mental Health Nursing.
Bulimia nervosa ( Eating Disorders) Mental Health Nursing.
 
SGK RỐI LOẠN KALI MÁU CỰC KỲ QUAN TRỌNG.pdf
SGK RỐI LOẠN KALI MÁU CỰC KỲ QUAN TRỌNG.pdfSGK RỐI LOẠN KALI MÁU CỰC KỲ QUAN TRỌNG.pdf
SGK RỐI LOẠN KALI MÁU CỰC KỲ QUAN TRỌNG.pdf
 
World-TB-Day-2023_Presentation_English.pptx
World-TB-Day-2023_Presentation_English.pptxWorld-TB-Day-2023_Presentation_English.pptx
World-TB-Day-2023_Presentation_English.pptx
 
"Radical excision of DIE in subferile women with deep infiltrating endometrio...
"Radical excision of DIE in subferile women with deep infiltrating endometrio..."Radical excision of DIE in subferile women with deep infiltrating endometrio...
"Radical excision of DIE in subferile women with deep infiltrating endometrio...
 
Basic structure of hair and hair growth cycle.pptx
Basic structure of hair and hair growth cycle.pptxBasic structure of hair and hair growth cycle.pptx
Basic structure of hair and hair growth cycle.pptx
 
ayurvedic formulations herbal drug technologyppt
ayurvedic formulations herbal drug technologypptayurvedic formulations herbal drug technologyppt
ayurvedic formulations herbal drug technologyppt
 
Trustworthiness of AI based predictions Aachen 2024
Trustworthiness of AI based predictions Aachen 2024Trustworthiness of AI based predictions Aachen 2024
Trustworthiness of AI based predictions Aachen 2024
 
Red Blood Cells_anemia & polycythemia.pdf
Red Blood Cells_anemia & polycythemia.pdfRed Blood Cells_anemia & polycythemia.pdf
Red Blood Cells_anemia & polycythemia.pdf
 
Physiotherapy Management of Rheumatoid Arthritis
Physiotherapy Management of Rheumatoid ArthritisPhysiotherapy Management of Rheumatoid Arthritis
Physiotherapy Management of Rheumatoid Arthritis
 
The Importance of Mental Health: Why is Mental Health Important?
The Importance of Mental Health: Why is Mental Health Important?The Importance of Mental Health: Why is Mental Health Important?
The Importance of Mental Health: Why is Mental Health Important?
 
How to cure cirrhosis and chronic hepatitis naturally
How to cure cirrhosis and chronic hepatitis naturallyHow to cure cirrhosis and chronic hepatitis naturally
How to cure cirrhosis and chronic hepatitis naturally
 

Angina.com slideshare march 24 2013

  • 1. Angina.com Angina.com March 24 2013 Research Interviews on Heart Disease Coronary Artery Disease and Angina.
  • 3. Medical Disclaimer | Terms and Conditions • The contents of the Hemodialysis.com Site, such as text, graphics, images, and other material contained on the Hemodialysis.com Site ("Content") are for informational purposes only. The Content is not intended to be a substitute for professional medical advice, diagnosis, or treatment. Always seek the advice of your physician or other qualified health provider with any questions you may have regarding a medical condition. Never disregard professional medical advice or delay in seeking it because of something you have read on the Hemodialysis.com Site! • If you think you may have a medical emergency, call your doctor or 911 immediately. Hemodialysis.com does not recommend or endorse any specific tests, physicians, products, procedures, opinions, or other information that may be mentioned on the Site. Reliance on any information provided by Hemodialysis.com or other Eminent Domains Inc (EDI) websites, EDI employees, others appearing on the Site at the invitation of Hemodialysis.com or EDI, or other visitors to the Site is solely at your own risk. • The Site may contain health- or medical-related materials that are sexually explicit. If you find these materials offensive, you may not want to use our Site. The Site and the Content are provided on an "as is" basis. Read more interviews on Hemodialysis.com
  • 4. Cardiac troponin I levels measured with a high-sensitive assay increase over time and are strong predictors of mortality in an elderly population. Angina.com Interview with Kai M Eggers, MD, PhD Department of Medical Sciences, Cardiology, Uppsala University Uppsala, Sweden • Angina.com: What are the main findings of the study ? • Dr. Eggers: One main finding was that troponin I levels in our elderly study participants increased significantly over five years time. Troponin is reflective of myocardial abnormalities and in fact, the most pronounced increases were seen in subjects with evidence of heart disease or risk factors for its development. Accordingly, we found that subjects with higher troponin levels at baseline or greater increases had a significant risk for death, even when other risk factors or biomarkers were taken into account. • Another important result of our study was that we for the first time were able to measure troponin I levels in almost all of our study participants. This depends on the analytical performance of the applied assay from Abbott and is at difference to other studies in general populations in whom troponin remained undetectable in at least every third subject. Angina.com: Were any of the findings unexpected ? • Dr. Eggers: We suspected that troponin I would be a strong risk predictor but were surprised to see that it remained independently prognostic in analyses that were adjusted for CRP and NT-proBNP, biomarkers that have received much greater attention in studies performed in community populations.
  • 5. Cardiac troponin I levels measured with a high-sensitive assay increase over time and are strong predictors of mortality in an elderly population. Angina.com Interview with Kai M Eggers, MD, PhD Department of Medical Sciences, Cardiology, Uppsala University Uppsala, Sweden (cont) • Angina.com: Were any of the findings unexpected ? • Dr. Eggers: We suspected that troponin I would be a strong risk predictor but were surprised to see that it remained independently prognostic in analyses that were adjusted for CRP and NT-proBNP, biomarkers that have received much greater attention in studies performed in community populations. • Angina.com: What should clinicians and patients take home from this study ? • Dr. Eggers:Troponins are usually measured in patients with the suspicion of a heart attack. We have learned from our study that troponins can also be detected in presumably healthy subjects. This must be taken into account when interpreting troponin results in elderly patients who are admitted because of chest pain. Another important point is that troponin levels appear to increase with aging as result of age-related changes of the heart. This is discussed more in detail in another paper from our group published this month in Clinical Chemistry, and must probably be considered when defining reference levels for troponin. • Angina.com: What further research do you recommend as a result of your study? • Dr. Eggers: I would like to see a study investigating whether lifestyle changes or medical interventions could influence troponin levels, and whether this also might reflect better outcome. This issue has for example been investigated in an interesting paper from deFilippi and colleagues published last year in JACC demonstrating that physical exercise was related to a smaller increase in troponin levels over time. If we could show that troponin levels would mirror the beneficial cardiac effects of this kind of interventions, then this would be a big step towards personalized preventive strategies. • Cardiac troponin I levels measured with a high-sensitive assay increase over time and are strong predictors of mortality in an elderly population. • Citation: • Eggers KM, Venge P, Lindahl B, Lind L. • Department of Medical Sciences, Uppsala University, Sweden. J Am Coll Cardiol. 2013 Mar 6. pii: S0735-1097(13)00936-4. doi: 10.1016/j.jacc.2012.12.048. [Epub ahead of print]
  • 6. Estimating the effect of long-term physical activity on cardiovascular disease and mortality: evidence from the Framingham Heart Study Angina.com Interview with Susan M. Shortreed PhD Assistant Investigator 
Biostatistics Unit, Group Health Research Institute Group Health Research InstituteSeattle, WA 98101 • Angina.com: What are the main findings of the study? • Dr. Shortreed: Physical activity throughout ones life is linked with longer life. • Angina.com: Were any of the findings unexpected? • Dr. Shortreed: The findings of this study confirmed that long-term physical activity has health benefits. • Our study followed individuals for a longer period of time than many previous studies in order to understand their physical activity habits and monitor their health outcomes. • Angina.com: What should clinicians and patients take away from this study? • Dr. Shortreed: Continued physical activity through out one’s life is important to a long life. Clinicians should talk with their patients about the benefits of physical activity and advise them on appropriate levels of physical activity considering their overall health. Patients should ask their clinician about the health benefits of physical activity and talk with their clinician before increasing their amount and intensity of physical activity. • The Centers for Disease Control and Prevention has some great online information and resources for both clinicians and patients on recommended physical activity levels for all ages.
  • 7. Estimating the effect of long-term physical activity on cardiovascular disease and mortality: evidence from the Framingham Heart Study Angina.com Interview with Susan M. Shortreed PhD Assistant Investigator 
Biostatistics Unit, Group Health Research Institute Group Health Research InstituteSeattle, WA 98101 (Cont) • Angina.com: What further research do you recommend as a result of your study? • Dr. Shortreed: We recommend studies investigate the long-term effects of physical activity in women and in diverse populations. • To investigate the effects of long-term physical activity, we recommend studies record information on activity levels over time and use appropriate analytic methods to quantify the link between long-term effects of physical activity and health benefits. • Citation: • Estimating the effect of long-term physical activity on cardiovascular disease and mortality: evidence from the Framingham Heart Study • Heart. 2013 Mar 8. [Epub ahead of print] • Shortreed SM, Peeters A, Forbes AB. Monash University, , Melbourne, Australia.
  • 8. Perioperative Dexmedetomidine Improves Outcomes of Cardiac Surgery Angina.com Interview with Hong Liu, MD Anesthesiology and Pain Medicine, University of California Davis Health System, 4150 V Street Suite 1200, Sacramento, CA, 95817 • Angina.com: What are the main findings of the study? • Dr. Liu: Perioperative dexmedetomidine use in cardiac surgical patients were more likely to have better in-hospital, 30-day and 1- year survival. The perioperative use of dexmedetomidine is also associated with a significant decrease in the incidence of postoperative overall complications and delirium. • There was no evidence of adverse hemodynamic side effects of dexmedetomidine in patients undergoing cardiac surgery. • Angina.com: Were any of the findings unexpected? • Dr. Liu: Our study did not find perioperative dexmedetomidine improving postoperative kidney function. • On the contrary, it worsened postoperative renal function.
  • 9. Perioperative Dexmedetomidine Improves Outcomes of Cardiac Surgery Angina.com Interview with Hong Liu, MD Anesthesiology and Pain Medicine, University of California Davis Health System, 4150 V Street Suite 1200, Sacramento, CA, 95817 (cont) • Angina.com: What should clinicians and patients take away from this study? • Dr. Liu: Dexmedetomidine used in clinical practice have many desirable effects including analgesia, anxiolysis, inhibition of central sympathetic outflow and reduction of systemic norepinephrine release that improve hemodynamic stability, positively affect myocardial oxygen supply and demand and may provide myocardial protection. It also increased urine output. If indicated, physicians should consider use it. • Because of the above-mentioned properties of this medication, patients may have bradycardia and polyuria right after surgery. • Angina.com: What further research do you recommend as a result of your study? • Dr. Liu: Multicenter randomized, prospective trial. Dose-dependent on kidney protection • Citation: • Perioperative Dexmedetomidine Improves Outcomes of Cardiac Surgery • Fuhai Ji, Zhongmin Li, Hung Nguyen, Nilas Young, Pengcai Shi, Neal Fleming, and Hong Liu • Circulation. 2013;CIRCULATIONAHA.112.000936published online before print March 19 2013, doi:10.1161/CIRCULATIONAHA.112.000936
  • 10. Effect of Intensive versus Moderate Lipid-Lowering Therapy on Epicardial Adipose Tissue in Hyperlipidemic Postmenopausal Women: A Substudy of the BELLES (Beyond Endorsed Lipid Lowering with EBT Scanning) Trial) Angina.com Interview with Paolo Raggi, MD, FACC, FACP, FASNC, FCCT Director, Mazankowski Alberta Heart Institute Professor of Medicine, University of Alberta Capital Health Chair in Cardiac Sciences Edmonton, AB T6G 2B7, Ca • Angina.com: What are the main findings of the study? • Dr. Raggi: The present study was a sub-analysis of the BELLES study, originally designed to assess the effect of intensive versus moderate lipid-lowering therapy on coronary artery calcium in post- menopausal women. • In the present sub-analysis, the effect on epicardial adipose tissue (EAT) was examined; EAT has recently emerged as a local source of inflammation and a surrogate and a possible active component in the pathogenesis of coronary atherosclerosis and of the formation of vulnerable atherosclerotic plaques. The main findings of the present sub-analysis are that although both treatment strategies resulted in halting the progression of EAT increase with age, only intensive lipid-lowering therapy was associated with a decrease in EAT volume.
  • 11. Effect of Intensive versus Moderate Lipid-Lowering Therapy on Epicardial Adipose Tissue in Hyperlipidemic Postmenopausal Women: A Substudy of the BELLES (Beyond Endorsed Lipid Lowering with EBT Scanning) Trial) Angina.com Interview with Paolo Raggi, MD, FACC, FACP, FASNC, FCCT Director, Mazankowski Alberta Heart Institute Professor of Medicine, University of Alberta Capital Health Chair in Cardiac Sciences Edmonton, AB T6G 2B7, Ca (cont) • Angina.com: Were any of the findings unexpected? • Dr. Raggi: The decrease in EAT volume was not associated with the decrease in LDL (bad) cholesterol. This mostly unexpected finding denotes that the beneficial effect of statin therapy on EAT is mediated through other mechanisms than LDL lowering; the most plausible mechanism is through the well-known anti-inflammatory effects of statins. • Angina.com: What should clinicians and patients take away from this study? • Dr. Raggi: This study provides another possible link between the beneficial effects of statins on cardiovascular risk; i.e. EAT volume modulation. It also provides possible evidence for a greater beneficial effect of intensive versus moderate lipid-lowering therapy. Finally, EAT may be used as surrogate of the effectiveness of risk-reduction therapies.
  • 12. Effect of Intensive versus Moderate Lipid-Lowering Therapy on Epicardial Adipose Tissue in Hyperlipidemic Postmenopausal Women: A Substudy of the BELLES (Beyond Endorsed Lipid Lowering with EBT Scanning) Trial) Angina.com Interview with Paolo Raggi, MD, FACC, FACP, FASNC, FCCT Director, Mazankowski Alberta Heart Institute Professor of Medicine, University of Alberta Capital Health Chair in Cardiac Sciences Edmonton, AB T6G 2B7, Ca • Angina.com: What further research do you recommend as a result of your study? • Dr. Raggi: A study of statin therapy with both EAT measurement pre- and post- treatment with adequate number of patients and follow-up to assess both surrogate (i.e. EAT volume change) and clinical events (major adverse cardiovascular events); if positive, this would establish the role of EAT volume measurement as a surrogate of clinical outcome. • Citation: • Effect of Intensive versus Moderate Lipid-Lowering Therapy on Epicardial Adipose Tissue in Hyperlipidemic Postmenopausal Women: A Substudy of the BELLES (Beyond Endorsed Lipid Lowering with EBT Scanning) Trial) • Alexopoulos N, Melek BH, Arepalli CD, Hartlage GR, Chen Z, Kim S, Stillman AE, Raggi P. • Division of Cardiology, Department of Medicine, Emory University, Atlanta, GA, USA. J Am Coll Cardiol. 2013 Mar 13. pii: S0735-1097(13)01002-4. doi: 10.1016/j.jacc.2012.12.051. [Epub ahead of print]
  • 13. Severity of coronary atherosclerosis in patients with a first acute coronary event: a diabetes paradox Angina.com Interview with Giampaolo Niccoli, MD, PhD, FESC Interventional Cardiologist Assistant professor of Cardiology Institute of Cardiology Catholic University of the Sacred Heart Rome, Italy • Angina.com: What are the main findings of the study? • Dr. Niccoli :In this study, we aimed at comparing coronary artery disease features at the time of a first acute coronary syndrome in type II diabetic and non-diabetic patients by coronary angiography and by optical coherence tomography. • In the angiographic substudy, diabetes was independently associated with both stenosis score and extent index. Furthermore, well developed collateral circulation toward the culprit vessel was more frequent in diabetic than in non-diabetic patients. In the optical coherence tomography substudy, at minimal lumen diameter site lipid quadrants were less and the lipid arc was smaller in diabetic than in non-diabetic patients. Furthermore, the most calcified cross-section along the culprit segment had a greater number of calcified quadrants and a wider calcified arc in diabetic than in non-diabetic patients. Superficial calcified nodules were more frequently found in diabetic than in non-diabetic patients. • Thus, in spite of potent pro-inflammatory, pro-oxidant and pro-thrombotic stimuli operating in type II diabetes, diabetic patients exhibit substantially more severe coronary atherosclerosis than non- diabetic patients at the time of a first acute coronary event. Better collateral development toward the culprit vessel, a predominantly calcific plaque phenotype and, probably, yet unknown protective factors operating in diabetic patients may explain these intriguing paradoxical findings.
  • 14. Severity of coronary atherosclerosis in patients with a first acute coronary event: a diabetes paradox Angina.com Interview with Giampaolo Niccoli, MD, PhD, FESC Interventional Cardiologist Assistant professor of Cardiology Institute of Cardiology Catholic University of the Sacred Heart Rome, Italy (cont) • Angina.com: Were any of the findings unexpected? • Dr. Niccoli : When investigating the association of coronary artery disease burden and diabetes status in an acute setting, we found a more severe and extensive coronary atherosclerosis as compared to non- diabetic patients, although pro- thrombotic and pro-inflammatory stimuli are stronger in diabetic than non- diabetic patients. • Angina.com What should clinicians and patients take away from this study? • Dr. Niccoli : As demonstrated in our study, at the time of their first acute coronary syndrome, type II diabetic patients have more severe and extensive coronary atherosclerosis as compared to non-diabetic patients. Thus, diabetic patients may have protective factors that prevent a first acute coronary syndrome. • Indeed, we also showed that type II diabetic patients had better collateral circulation towards the culprit vessel than non-diabetic patients, Therefore, physicians should consider this apparent contradiction which is probably accounted for by the fact that diabetes offsets the detrimental effects of a high extent index on collateral development. Furthermore, a more calcific atherosclerosis may stabilize coronary plaque.
  • 15. Severity of coronary atherosclerosis in patients with a first acute coronary event: a diabetes paradox Angina.com Interview with Giampaolo Niccoli, MD, PhD, FESC Interventional Cardiologist Assistant professor of Cardiology Institute of Cardiology Catholic University of the Sacred Heart Rome, Italy (cont) • Angina.com: What further research do you recommend as a result of your study? • Dr. Niccoli : Further studies will probably identify other, and possibly more important, causes of the paradox that in diabetic patients the first acute event occurs in the presence of more severe coronary artery disease. Interestingly, protective effect mediated by inhibition of matrix metalloproteinase-9, which reduces elastin degradation might operate in the coronary circulation and should be investigated. • Citation: • Severity of coronary atherosclerosis in patients with a first acute coronary event: a diabetes paradox. • Eur Heart J. 2013 Mar;34(10):729-41. doi: 10.1093/eurheartj/ehs393. Epub 2012 Nov 27. • Niccoli G, Giubilato S, Di Vito L, Leo A, Cosentino N, Pitocco D, Marco V, Ghirlanda G, Prati F, Crea F. • Institute of Cardiology, Catholic University of the Sacred Heart, Largo F. Vito 1, 00168 Rome, Italy.
  • 16. Sexual Function After Left Ventricular Assist Device Angina.com Author Interview: Peter Eckman, M.D. Department of Medicine Cardiovascular Division, University of Minnesota Medical School, 420 Delaware Street S.E., MMC 508, Minneapolis, MN 55455 • Angina.com: What are the main findings of the study? • Dr. Eckman: We found that 71% of men and 79% of women with a continuous flow left ventricular assist device (LVAD) met criteria for sexual dysfunction, as measured by the Changes in Sexual Function Questionnaire-14. Furthermore, more than half of the survey respondents noted worsening in arousal and orgasm. Age over 60 and history of ischemic heart disease were directly associated with sexual dysfunction. Psychological issues, including fear of damaging the LVAD, self- harm, device alarms, and hurting one’s partner were also cited, and most noted that the equipment was “somewhat” or a “major” obstacle to sexual satisfaction. Angina.com: Were any of the findings unexpected? • Dr. Eckman: LVADs have been shown to significant improve quality of life, so the relatively poorer sexual function was surprising. We were not surprised that older age and history of ischemic heart disease were associated with worse function. One participant said “Probably the least improved part of life before to after VAD.” On the other hand, we were encouraged to see that a notable minority of patients reported improvements in sexual function. We were also surprised that only a minority of patients reported improvement in desire.
  • 17. Sexual Function After Left Ventricular Assist Device Angina.com Author Interview: Peter Eckman, M.D. Department of Medicine Cardiovascular Division, University of Minnesota Medical School, 420 Delaware Street S.E., MMC 508, Minneapolis, MN 55455 (cont) • Angina.com: What should clinicians and patients take away from this study? • Dr. Eckman: This data should inform subsequent studies designed to evaluate and improve understanding of the specific factors involved in sexual health after LVAD implant. Equipment was often cited as an obstacle, and improvements to external components may have a favorable impact on sexual function. Willingness to address concerns about sexual function may help minimize barriers to sexual activity and satisfaction after LVAD • Angina.com: What further research do you recommend as a result of your study? • Dr. Eckman: It is unclear if our findings were driven by physiologic changes such as altered blood flow and endothelial function, or any of the myriad other factors that impact sexual function. Study of the physiology of genital blood flow after implant of continuous-flow LVAD may provide additional insight into the relative importance of pathophysiology on sexual health after LVAD. • Citation: • Sexual Function After Left Ventricular Assist Device • Eckman PM, Dhungel V, Mandras S, et al. J Am Coll Cardiol. 2013;():. doi:10.1016/j.jacc.2013.02.022.
  • 18. Yield and Diagnostic Value of Stress Myocardial Perfusion Imaging in Patients Without Known Coronary Artery Disease Presenting with Syncope Angina.com Interview with Wael A. AlJaroudi MD Division of Cardiovascular Disease American University of Beirut Medical Center Riad El-Solh, Beirut 1107 2020, Lebanon • Angina.com: What are the main findings of the study? • Answer: In this study, we investigated the diagnostic yield of stress myocardial perfusion imaging (MPI) for patients presenting for syncope without prior history of known coronary artery disease (CAD). Out of 700 patients, 659 (94%) had normal perfusion. Of the 41 patients with abnormal MPI, 18 had a coronary angiogram and half of them were falsely positive. The diagnostic yield of the stress MPI (95% CI) was 0.88% (0.3%, 2.6%), 1.3%(0.3%, 4.0%) and 2.4% (0.61%, 7.3%) for patients with low, intermediate and high risk categories based on Framingham scores, respectively. Also, an abnormal stress MPI did not have prognostic value, in part due to the low numbers of abnormal studies and high false- positive results. • As such, stress MPI for evaluation of syncope in patients without known CAD has a low diagnostic yield among all risk groups, and reaffirmation or revision of the appropriateness criteria should be considered.
  • 19. Yield and Diagnostic Value of Stress Myocardial Perfusion Imaging in Patients Without Known Coronary Artery Disease Presenting with Syncope Angina.com Interview with Wael A. AlJaroudi MD Division of Cardiovascular Disease American University of Beirut Medical Center Riad El-Solh, Beirut 1107 2020, Lebanon (cont) • Angina.com: Were any of the findings unexpected? • Answer: Based on our clinical experience, we were expecting a low diagnostic yield for stress MPI in such group of patients. However, there are limited data that justify using stress MPI to rule out ischemia as a cause of syncope in patients without known history of CAD. Given the high economic burden and radiation exposure from these tests, we sought to investigate whether indeed the diagnostic yield is high enough to justify using these test or whether the appropriateness criteria needs revisiting. • Angina.com: What should clinicians and patients take away from this study? • Answer: In patients without known CAD, syncope is not an anginal equivalent, and therefore the diagnostic yield of stress MPI is very low regardless of the Framingham score. Perhaps exercise stress echocardiogram might have better since it looks at structure, dynamic changes, and valves. This needs to be further studied and evaluated.
  • 20. Yield and Diagnostic Value of Stress Myocardial Perfusion Imaging in Patients Without Known Coronary Artery Disease Presenting with Syncope Angina.com Interview with Wael A. AlJaroudi MD Division of Cardiovascular Disease American University of Beirut Medical Center Riad El-Solh, Beirut 1107 2020, Lebanon (cont) • Angina.com: What further research do you recommend as a result of your study? • Answer: It would be interesting to evaluate the diagnostic yield of stress MPI for patients with known coronary artery disease and/or ischemic cardiomyopathy presenting with syncope. • Citation: • Yield and Diagnostic Value of Stress Myocardial Perfusion Imaging in Patients Without Known Coronary Artery Disease Presenting with Syncope • Wael A. AlJaroudi, M. Chadi Alraies, Oussama Wazni, Manuel D. Cerqueira, and Wael A. Jaber • Circ Cardiovasc Imaging. 2013;CIRCIMAGING.112.000051published online before print February 15 2013, doi:10.1161/CIRCIMAGING.112.000051
  • 21. Renoprotective effect of remote ischemic postconditioning by intermittent balloon inflations in patients undergoing percutaneous coronary intervention Angina.com Interview with Spyridon Deftereos, MD; Georgios Giannopoulos, MD University of Athens, Athens, Greece • Angina.com: What are the main findings of the study? • Answer: The principal finding of this study is that a procedure of repeated inflations and deflations of the stent balloon used to perform PCI (percutaneous coronary intervention) in patients with non-ST-elevation myocardial infarction leads to a reduction in the periprocedural acute kidney injury rate. This effect seems to have a clinical impact, as it was accompanied by a reduction (of borderline statistical significance) in the 30-day rate of death or rehospitalization for any cause. • The physiological process underlying this procedure is termed “remote ischemic conditioning” (“remote” because the conditioning stimulus is applied to a site far away from the organ that is protected against tissue damage; in our case, the conditioning stimulus is applied to the heart and the conditioned organ is the kidney) and, although a lot of unanswered questions remain as to its mechanisms, experimental and clinical evidence has repeatedly demonstrated its existence.
  • 22. Renoprotective effect of remote ischemic postconditioning by intermittent balloon inflations in patients undergoing percutaneous coronary intervention Angina.com Interview with Spyridon Deftereos, MD; Georgios Giannopoulos, MD University of Athens, Athens, Greece (cont) • Angina.com: Were any of the findings unexpected? • Answer: Actually no. The study results could have gone either way; that is a negative result would not have been a surprise either. However, we did have a smaller retrospective study published a couple of years ago suggesting that remote ischemic conditioning could be effective in this setting. Our present results show that that hypothesis-generating study pointed to the right direction. • Angina.com: What should clinicians and patients take away from this study? • Answer: An important point associated with this study is that cardiac interventionalists should always “remember the kidney” when performing cardiac catheterizations, especially in patients with acute coronary syndromes (since these patients are more vulnerable to acute renal damage compared to those undergoing elective procedures). Effective measures of renoprotection, including appropriate hydration, withholding of nephrotoxic drugs, parsimonious use of contrast and, probably (if our results are confirmed by further clinical trials), remote ischemic conditioning should be employed to reduce the likelihood of renal damage.
  • 23. Renoprotective effect of remote ischemic postconditioning by intermittent balloon inflations in patients undergoing percutaneous coronary intervention Angina.com Interview with Spyridon Deftereos, MD; Georgios Giannopoulos, MD University of Athens, Athens, Greece (cont) • Angina.com: What further research do you recommend as a result of your study? • Answer: Our positive results, along with other recent ones, should prompt a larger clinical study, powered enough to demonstrate clinical efficacy, if any, in terms of hard clinical endpoints. • Our study was underpowered to have a primary clinical endpoint, but the secondary finding of a reduction in the 30-day clinical event rate is certainly encouraging. • Citation: • Renoprotective effect of remote ischemic postconditioning by intermittent balloon inflations in patients undergoing percutaneous coronary intervention • Deftereos S, Giannopoulos G, Tzalamouras V, Raisakis K, Kossyvakis C, Kaoukis A, Panagopoulou V, Karageorgiou S, Avramides D, Toutouzas K, Hahalis G, Pyrgakis V, Manolis AS, Alexopoulos D, Stefanadis C, Cleman MW. • Department of Cardiology, Athens General Hospital “G. Gennimatas”, Athens, Greece. • J Am Coll Cardiol. 2013 Mar 13. pii: S0735-1097(13)01001-2. doi: 10.1016/j.jacc.2013.02.023. [Epub ahead of print]
  • 24. Duration of Ventilations during Cardiopulmonary Resuscitation by Lay-Rescuers and First Responders: Relationship between Delivering Chest Compressions and Outcomes Angina.com Interview with S.G. Beesems, MSc afdeling Cardiologie Kamer G4-248 Academisch Medisch Centrum 1105 AZ Amsterdam • Angina.com: What are the main findings of the study? • Answer: The results of our study show that lay-rescuers require a median ventilation time of 7 seconds to complete two ventilation attempts. Only 21% are able to fully meet the ventilation guidelines of 2010. However, 97% of all rescuers provided chest compressions above the recommended minimum of 60 chest compressions in one minute, 88% of all administered over 70 chest compressions in one minute and 63% of all administered over 80 chest compressions in one minute. Hospital survival of patients with all rhythms was 25%. Therefore, the objective of delivering sufficient chest compressions is met by all but a few rescuers. • Angina.com: Were any of the findings unexpected? • Answer: In our study, better survival was observed with longer ventilations. This is against the findings of other studies with long interruptions of chest compressions, mainly caused by pauses associated with defibrillation shocks. This paradox can be attributed to the fact that rescuers who had the longest pauses for ventilations also arrived and shocked earlier with the AED. After adjustment for the baseline factors, longer pauses for ventilation were not associated with worse survival. It is also possible that the suggested detrimental effect of perishock pauses does not apply to pauses for ventilation
  • 25. Duration of Ventilations during Cardiopulmonary Resuscitation by Lay-Rescuers and First Responders: Relationship between Delivering Chest Compressions and Outcomes Angina.com Interview with S.G. Beesems, MSc afdeling Cardiologie Kamer G4-248 Academisch Medisch Centrum 1105 AZ Amsterdam (cont) • Angina.com: What should clinicians and patients take away from this study? • Answer: Lay-rescuers with CPR training that includes chest compressions and ventilations are able to perform CPR better than previously believed. Longer pauses for ventilations are not associated with worse outcome. Guidelines may allow longer pauses for ventilations without detriment to survival. • Citation: • Duration of Ventilations during Cardiopulmonary Resuscitation by Lay-Rescuers and First Responders: Relationship between Delivering Chest Compressions and Outcomes. • Beesems SG, Wijmans L, Tijssen JG, Koster RW. • 1Academic Medical Center, Amsterdam, The Netherlands. Circulation. 2013 Mar 18. [Epub ahead of print]
  • 26. Troponin I, but not BNP, is Associated with Phosphorus, Calcium and Vitamin D in Stable Coronary Artery Disease Angina.com Interview with José Pedro L. Nunes M.D. Associate professor at University of Porto, Portugal. • Angina.com: What are the main findings of the study? • Dr. Nunes: Elevated plasma cardiac troponin, elevated plasma phosphorus and decreased plasma vitamin D are known to be associated with negative outcomes. In this study, data was obtained from a relatively small number of patients (sixty) with stable coronary artery disease and preserved left ventricular function, and patients with higher values for cardiac troponin I were shown to have higher plasma phosphorus, higher plasma calcium and lower plasma 25-OH vitamin D values when compared to patients with lower values for troponin. The same association was not found with BNP. • Angina.com: Were any of the findings unexpected? • Dr. Nunes: When you start an investigation, you do not know what you will find, but previous studies, carried out in renal patients, had shown an association between troponin levels and FGF-23, which itself is associated to phosphorus.
  • 27. Troponin I, but not BNP, is Associated with Phosphorus, Calcium and Vitamin D in Stable Coronary Artery Disease Angina.com Interview with José Pedro L. Nunes M.D. Associate professor at University of Porto, Portugal. (cont) • Angina.com: What should clinicians and patients take away from this study? • Dr. Nunes: Given the relatively small number of patients under study, the conclusions should be seen with some caution. In any event, troponin is no longer seen solely as a marker for myocardial infarction. Cardiac/renal connections could find here yet another important field of future research. As stated in the text, “the hypothesis is raised that deleterious effects of higher phosphorus and/or calcium and of lower vitamin D levels on the heart are reflected by an increase in plasma troponin I”. • Angina.com: What recommendations do you have for future research as a result of your study? • Dr. Nunes: It would be important to try to confirm these findings in larger and different sets of patients. If confirmed, these findings could be a starting point for significant advances in the understanding of mechanisms of cardiac injury, perhaps involving diet, lack of sunlight exposure and with a possible impact on vitamin D supplementation. • Citation: • Troponin I, but not BNP, is Associated with Phosphorus, Calcium and Vitamin D in Stable Coronary Artery Disease • Nunes J.P.L. • Faculdade de Medicina da Universidade do Porto: Center for Research in Health Technologies and Information Systems (Cintesis), Porto • Kidney Blood Press Res 2013;37:43-47 (DOI:10.1159/000343399) • Published online first (Issue-in-Progress)
  • 28. Primary Percutaneous Coronary Intervention in Patients With Acute Myocardial Infarction, Resuscitated Cardiac Arrest, and Cardiogenic Shock : The Role of Primary Multivessel Revascularization Angina.com Interview with Darren Mylotte MB BCh MD ICPS – Institut Cardiovasculaire Paris Sud · Interventional Cardiology • Angina.com: What are the main findings of the study? • Dr. Mylotte: The study demonstrates the very high mortality rate of patients with STEMI, cardiogenic shock and resuscitated cardiac arrest: 65.4% at 6-months. It also demonstrates that those with more extensive myocardial ischemia due to more widespread coronary disease have worse outcomes compared to those with less ischemia. Furthermore, it demonstrates that in those with multivessel coronary disease, multivessel PCI appears to improve survival compared to culprit-only PCI. Finally, successful PCI and multivessel PCI were independent predictors of 6-month survival in those with multivessel disease.
  • 29. Primary Percutaneous Coronary Intervention in Patients With Acute Myocardial Infarction, Resuscitated Cardiac Arrest, and Cardiogenic Shock : The Role of Primary Multivessel Revascularization Angina.com Interview with Darren Mylotte MB BCh MD ICPS – Institut Cardiovasculaire Paris Sud · Interventional Cardiology (cont) • Angina.com: Were any of the findings unexpected? • Dr. Mylotte: The fact that multivessel primary PCI was associated with improved survival was not a surprise. Indeed the study hypothesis stemmed from other data suggesting enhanced outcomes in this critically-ill patient population. However, the magnitude of the survival advantage (absolute survival advantage 23.5%) was unexpected. While this may also reflect some selection or treatment bias, there is a clear signal that in selected cases multivessel PCI is advantageous. • Angina.com: What should clinicians and patients take away from this study? • Dr. Mylotte: I think that the central message of this study is that in patients with multivessel disease and cardiogenic shock, refractory to culprit-artery PCI, that further revascularization should be considered.
  • 30. Primary Percutaneous Coronary Intervention in Patients With Acute Myocardial Infarction, Resuscitated Cardiac Arrest, and Cardiogenic Shock : The Role of Primary Multivessel Revascularization Angina.com Interview with Darren Mylotte MB BCh MD ICPS – Institut Cardiovasculaire Paris Sud · Interventional Cardiology (cont) • Angina.com: What further research do you recommend as a result of your study? • Dr. Mylotte: I think that this study highlights the need for further randomized trials comparing revascularization strategies in this very high-risk group of patients. Furthermore, it demonstrates the limitations of coronary revascularization and that other treatments need to be developed in order to significantly improve outcomes. • Citation: • Primary percutaneous coronary intervention in patients with acute myocardial infarction, resuscitated cardiac arrest, and cardiogenic shock: the role of primary multivessel revascularization. • Mylotte D, Morice MC, Eltchaninoff H, Garot J, Louvard Y, Lefèvre T, Garot P. • Institut Cardiovasculaire Paris Sud, Institut Hospitalier Jacques Cartier, Massy, and Hôpital Claude Galien, Quincy, France. JACC Cardiovasc Interv. 2013 Feb;6(2):115-25. doi: 10.1016/j.jcin.2012.10.006. Epub 2013 Jan 23.
  • 31. Genetic Determinants of Dabigatran Plasma Levels and Their Relation to Bleeding. Angina.com Interview with Guillaume Paré MD, MSc, FRCPC Assistant ProfessorPathology and Molecular Medicine (primary) Clinical Epidemiology & Biostatistics Canada Research Chair in Genetic and Molecular Epidemiology Director, Genetic and Molecular Epidemiology Laboratory Visiting Scientist, Harvard Medical School McMaster University • Angina.com: What are the main findings of the study? • Dr. Pare: We conducted a genome-wide association study of dabigatran peak and trough concentration in 2,944 RE-LY participants with both genetic and pharmacokinetic data. The CES1 SNP rs2244613 was associated with a 15% decrease in trough concentrations per allele, and the ABCB1 SNP rs4148738 and CES1 SNP rs8192935 were associated with peak concentrations at genome-wide significance (P<9 x 10-8), also with a gene-dose effect. CES1 encodes for the liver carboxylesterase 1 enzyme, an esterase responsible for the biotransformation of dabigatran etexilate into the active metabolite, dabigatran. The ABCB1 gene encodes for P-glycoprotein, an ATP-dependent drug efflux pump for xenobiotic compounds with broad substrate specificity. Dabigatran etexilate, but not dabigatran, is an ABCB1 substrate and ABCB1 inhibitors increase dabigatran bioavailability by 10-20%. When tested for association with bleed, the CES1 SNP rs2244613 was associated with a lower risk of any bleeding (OR=0.67, 95%CI 0.55-0.82; P=7 x 10-5) in dabigatran-treated participants, and with a consistent but non-significant lower risk of major bleeding (OR=0.66, 95%CI 0.43-1.01). Importantly, the interaction between treatment (warfarin versus all dabigatran) and carrier status was statistically significant (P=0.002) with carriers having less bleeding with dabigatran than warfarin (HR=0.59, 95%CI 0.46-0.76; P=5.2 x 10-5) in contrast to no difference in noncarriers (HR=0.96, 95%CI 0.81-1.14; P=0.65).
  • 32. Genetic Determinants of Dabigatran Plasma Levels and Their Relation to Bleeding. Angina.com Interview with Guillaume Paré MD, MSc, FRCPC Assistant ProfessorPathology and Molecular Medicine (primary) Clinical Epidemiology & Biostatistics Canada Research Chair in Genetic and Molecular Epidemiology Director, Genetic and Molecular Epidemiology Laboratory Visiting Scientist, Harvard Medical School McMaster University (cont) • Angina.com: Were any of the findings unexpected? • Dr. Pare: While the two gene loci identified were to be expected given dabigatran etexilate’s metabolism, we were surprised by the strength of the genetic effects, especially on bleeds. Also, dabigatran etexilate was thought to be converted into dabigatran completely and immediately by esterases. Identification of a genetic variant of CES1 associated with drug exposure suggests that biotransformation into dabigatran could be a rate-limiting factor in certain individuals. • Angina.com: What should clinicians and patients take away from this study? • Dr. Pare: These results are significant as the relative risk of bleeding was 0.73 (95%CI 0.63-0.86) for the 32.8% of RE-LY participants that are rs2244613 minor allele carriers versus noncarriers, whereas the reported relative risk of bleeding was 0.86 (95% CI 0.81-0.93) for the lower (110 mg bid) versus higher dose (150 mg bid) of dabigatran in the overall study. The observed genetic effect was thus larger than the effect of drug dosage in the parent study, suggesting a potential role of genetics to help determine the optimal dose of dabigatran etexilate in individual patients. For example, carriers of the protective allele could perhaps be safely prescribed the higher dose of dabigatran etexilate even if elderly.
  • 33. Genetic Determinants of Dabigatran Plasma Levels and Their Relation to Bleeding. Angina.com Interview with Guillaume Paré MD, MSc, FRCPC Assistant ProfessorPathology and Molecular Medicine (primary) Clinical Epidemiology & Biostatistics Canada Research Chair in Genetic and Molecular Epidemiology Director, Genetic and Molecular Epidemiology Laboratory Visiting Scientist, Harvard Medical School McMaster University (cont) • Angina.com: What further research do you recommend as a result of your study? • Dr. Pare: Many key questions remain to be addressed. Among them is whether the CES1 SNP rs2244613 is associated with ischemic events as well as bleeds. Indeed, despite the large sample size, only 66 dabigatran etexilate-treated individuals suffered from any ischemic event in the genetic analysis sub-group of the RE-LY trial and power to detect a genetic association was accordingly limited. Larger studies will therefore be necessary to properly answer this question. Finally, clinical utility of CES1 genotyping will need to be assessed through rigorous studies carefully evaluating the health benefit and cost of incorporating genotypes into dose selection decisions. • Citation: • Genetic Determinants of Dabigatran Plasma Levels and Their Relation to Bleeding. • Paré G, Eriksson N, Lehr T, Connolly S, Eikelboom J, Ezekowitz MD, Axelsson T, Haertter S, Oldgren J, Reilly P, Siegbahn A, • Syvänen AC, Wadelius C, Wadelius M, Zimdahl-Gelling H, Yusuf S, Wallentin L. 1Population Health Research Institute, Hamilton Health Sciences and McMaster University, Hamilton, Canada. Circulation. 2013 Mar 6. [Epub ahead of print]
  • 34. Rivaroxaban Reduces Spontaneous and Large Myocardial Infarctions: Findings from the ATLAS ACS 2 – TIMI 51 Trial Angina.com Interview with Matthew A. Cavender, M.D. Research Fellow, TIMI Study Group Brigham and Women’s Hospital Harvard Medical School Boston, Massachusetts 02115 • Angina.com: What are the main findings of the study? • Dr. Cavender: The ATLAS ACS 2 – TIMI 51 trial showed that treatment with rivaroxaban reduces cardiovascular death, myocardial infarction and stroke. To better understand how this drug improves cardiovascular outcomes, it was important to understand the effect of the rivaroxaban on specific types of myocardial infarctions. • Angina.com: Were any of the findings unexpected? • Dr. Cavender: In the overall trial, rivaroxaban 2.5 mg twice daily reduced cardiovascular death, myocardial infarction, or stroke. In addition, rivaroxaban 2.5 mg was shown to reduce cardiovascular death. The present study offers insight into the possible mechanisms leading to this mortality benefit since rivaroxaban reduces both spontaneous and large myocardial infarctions.
  • 35. Rivaroxaban Reduces Spontaneous and Large Myocardial Infarctions: Findings from the ATLAS ACS 2 – TIMI 51 Trial Angina.com Interview with Matthew A. Cavender, M.D. Research Fellow, TIMI Study Group Brigham and Women’s Hospital Harvard Medical School Boston, Massachusetts 02115 (cont) • Angina.com: What should clinicians and patients take away from this study? • Dr. Cavender: Our analysis showed the majority of myocardial infarctions in the ATLAS ACS 2 – TIMI 51 trial occurred spontaneously due to plaque rupture and coronary artery thrombosis. Rivaroxaban was given to patients to prevent these types of thrombotic events. We found that treatment with rivaroxaban significantly reduced spontaneous myocardial infarctions by approximately 20%, and both the 2.5 and 5 mg doses had directionally similar effects. • Citation: • Presented at 2013 ACC American College of Cardiology March 2013 Rivaroxaban Reduces Spontaneous and Large Myocardial Infarctions: Findings from the ATLAS ACS 2 – TIMI 51 Trial
  • 36. Value of Myocardial Perfusion Imaging in Renal Transplant Evaluation Angina.com Interview with: Dr. Chong Ghee Chew Department of Nuclear Medicine, PET and Bone Mineral Densitometry, Royal Adelaide Hospital, Adelaide, SA • Angina.com: What are the main findings of the study? • Answer: This is a retrospective audit of the cardiac outcomes of renal failure patients who had been transplanted in South Australia between 1999 to 2009, who had myocardial perfusion SPECT scan for the transplant assessment. The results represent ”real world” outcomes as the scans were performed in the 3 major teaching hospitals in SA. 2 endpoints – “soft” = inpatient care with angina +/- PCI +/- CABG, and “hard” = inpatient care with myocardial infarction or cardiac death. With a negative scan this cohort had a statistically significant lower soft endpoint event rate than a positive scan …3.9% vs 20.8%, hazard ratio of 4.4 at 5 years post scan. The hard endpoint event rate was also lower for those with a negative scan but the difference did not reach statistical significance. The event rates of hard and soft endpoints were no different for the negative scans that were performed with a tachycardic stress (treadmill exercise, dobutamine or external wire right atrial pacing) versus dipyridamole induced coronary vasodilatation.
  • 37. Value of Myocardial Perfusion Imaging in Renal Transplant Evaluation Angina.com Interview with: Dr. Chong Ghee Chew Department of Nuclear Medicine, PET and Bone Mineral Densitometry, Royal Adelaide Hospital, Adelaide, SA (cont) • Angina.com: Were any of the findings unexpected? • Answer: No • Angina.com: What should clinicians and patients take away from this study? • Answer: Myocardial perfusion SPECT scan is a good predictor of cardiac events in renal failure patients who are being considered for transplantation. • This is a valid test for transplant assessment. • Angina.com: What further research do you recommend as a result of your study? • The study did not include patients who were assessed but were not transplanted. We are planning another similar audit to look at this cohort. • Citation: • ACC 2013 American College Cardiology Presentation Spring 2013
  • 38. Angiographic and clinical characteristics of type 1 versus type 2 perioperative myocardial infarction Angina.com Interview with: Ivan Hanson, M.D. Chief Interventional Cardiology Fellow Beaumont Hospital, Royal Oak, MI • Angina.com: What are the main findings of the study? • Dr. Hanson: Over half of patients with perioperative myocardial infarction (PMI) have invasive angiographic evidence of plaque rupture (type 1 PMI). Patients with type 1 PMI had more intraoperative hypotension, higher incidence of transmural ischemia, larger infarct size and trended toward worse outcome. • Angina.com: Were any of the findings unexpected? • Dr. Hanson: Our hypothesis was that the majority of PMI events were due to hemodynamic insults in the setting of stable, but severely stenotic, coronary artery stenoses (type 2 PMI). • Not only did we observe a higher incidence of type 1 PMI than type 2 PMI, but type 1 PMI patients actually had greater incidence of intraoperative hypotension. This challenges the notion that hemodynamic pertubations are more closely associated with type 2 PMI. • Furthermore, clinical assessment alone, including knowledge of intraoperative hemodynamics and postoperative ECG findings, are insufficient to diagnose type of PMI.
  • 39. Angiographic and clinical characteristics of type 1 versus type 2 perioperative myocardial infarction Angina.com Interview with: Ivan Hanson, M.D. Chief Interventional Cardiology Fellow Beaumont Hospital, Royal Oak, MI (cont) • Angina.com: What should clinicians and patients take away from this study? • Dr. Hanson: Our findings suggest that if a sophisticated understanding of type of PMI is desired, invasive angiography may be useful. • Patients that were formally assumed to have type 2 PMI may in fact have plaque rupture and coronary thrombosis, which would not be expected to resolve by optimization of coronary blood flow alone. • Angina.com: What further research do you recommend as a result of your study? • Dr. Hanson: To test the hypothesis that tailoring management of PMI based on angiographic findings results in optimal outcomes, a much larger study is required. • I am also very interested in the invasive imaging characteristics of plaques that lead to, or have caused, PMI. • Citation: • Angiographic and clinical characteristics of type 1 versus type 2 perioperative myocardial infarction.
  • 40. Centralized, Stepped, Patient Preference–Based Treatment for Patients With Post–Acute Coronary Syndrome Depression Angina.com Interview with Karina W. Davidson, PhD Center for Behavioral Cardiovascular Health, Department of Medicine, Columbia University Medical Center, New York, New York • Angina.com: What are the main findings of the study? • Dr. Davidson: With a grant from the National Institutes of Health’s National Heart, Lung, and Blood Institute (NHLBI), Karina W. Davidson, PhD, director of the Center for Behavioral Cardiovascular Health at Columbia University Medical Center (CUMC), together with her colleagues at multiple sites in the U.S., completed a randomized controlled trial with 150 patients with elevated depressive symptoms two to six months after their hospitalization for heart disease. Patients were recruited from seven centers across the United States and were randomized to receive either six months of their preferred depression care (therapy, antidepressants or both) or to receive depression care determined by their physician upon notification that the patient had elevated depressive symptoms. • In the self-selected group, patients were given a choice of whether they wanted therapy or antidepressants, or both. “We tried to help them really think through what does it take to take a pill every day; what does it take to talk to a person about painful feelings and do homework, and really consider what will fit into their life,” said Dr. Davidson. “And every six to eight weeks, we revisited how their chosen treatment was working for them, and helped them make adjustments based upon their feedback.”
  • 41. Centralized, Stepped, Patient Preference–Based Treatment for Patients With Post–Acute Coronary Syndrome Depression Angina.com Interview with Karina W. Davidson, PhD Center for Behavioral Cardiovascular Health, Department of Medicine, Columbia University Medical Center, New York, New York (cont) • Therapy in this study was given virtually (by phone or web-based) to ensure that patients had access to clinicians of a similar skill set – so two counselors (one at CUMC and another at Yale) provided the therapy in this study. The virtual therapy also helped connect patients with therapists at times that were most convenient for them. “Maybe 8 p.m. at your home is the right time for you, but not 8 p.m. if you have to go for a clinic or hospital and wait half an hour and travel back home afterwards, etc.,” said Dr. Davidson, who is also professor of behavioral medicine in medicine, cardiology, and psychiatry, CUMC. The therapy used in the trial was problem solving treatment (PST) – a practical, hands-on approach that asks patients to choose the problems in their life that they want to tackle, teaches them how to problem solve, and then they go out and do homework to try the solutions and come back and talk about what worked, what didn’t, and tackle the next problem. • Findings showed that depressive symptoms were reduced more in the patient-selected, centralized depression care group, than in the patients receiving usual care. At the start of the trial, patients had an average of 19 points on the BDI depression scale – at the end of the trial, BDI scores in the patient-selected group were significantly reduced by an average of 3-6 points more than the reduction that happened in the usual care group. Getting down below 10 on the BDI puts one in the normal range – and almost half of the patients who received patient- selected, centralized treatment get down to the normal range; whereas only one-third in the usual care group got down to the normal range. • Importantly, researchers found that running centralized depression care at seven sites nationwide was effective, suggesting that it is feasible to deliver depression care to those who don’t have specialists in their area. Overall, the total health care costs for the self-selected group was $325 less than the control group, even when factoring in the cost of providing them with the therapy. The centralized depression care group demonstrated a trend towards fewer hospitalizations, compared to the usual care group.
  • 42. Centralized, Stepped, Patient Preference–Based Treatment for Patients With Post–Acute Coronary Syndrome Depression Angina.com Interview with Karina W. Davidson, PhD Center for Behavioral Cardiovascular Health, Department of Medicine, Columbia University Medical Center, New York, New York (cont) • Angina.com: Were any of the findings unexpected? • Dr. Davidson: Most depression trials in this area have gotten effect size of .3, which is one-third of the standard deviation. And we got .6, or 2/3 of the standard deviation. So we had almost double the size of treatment improvement compared to other people. • The research team believes that the difference in their findings can be explained in the difference between how previous published trials were conducted, versus how their trial was conducted. In previous trials, patients were often offered either one type of treatment or usual care, and they had to remain with that selection for the remainder of the trial. However, because it’s well known that there isn’t one magic bullet for depression; one thing doesn’t work for all patients, the CODIACS Vanguard trial was designed with a flexible algorithm that allowed the clinical team to regularly liaise with the patients to help them keep different treatment options and combinations, until one was found that suited them best. • Angina.com: What should clinicians and patients take away from this study? • Dr. Davidson: About 1.2 million Americans survive an ACS event every year and many of them have clinically significant and persistent depression. Post-ACS depression is also associated with an increased risk of ACS recurrence and with an increase in the relative risk of all-cause mortality, the authors write in the study background. These findings show that treating heart attack survivors for depressive symptoms is feasible, effective, and even has the tantalizing possibility of being cost- neutral. Importantly, treating depressive symptoms—as opposed to a diagnosis of depression— should be determined by an alliance between a healthcare professional and the patient – and regularly assessed for effectiveness and altered to continue to meet the patient’s needs.
  • 43. Centralized, Stepped, Patient Preference–Based Treatment for Patients With Post–Acute Coronary Syndrome Depression Angina.com Interview with Karina W. Davidson, PhD Center for Behavioral Cardiovascular Health, Department of Medicine, Columbia University Medical Center, New York, New York (cont) • Angina.com: What further research do you recommend as a result of your study?Dr. Davidson: Dr. Karina Davidson and her team plan to explore whether these results can be replicated in a much larger nationwide randomized controlled trial, to examine whether treating them for depressive symptoms lowers their risk of death or having another heart attack. • Citation: • Centralized, Stepped, Patient Preference–Based Treatment for Patients With Post– Acute Coronary Syndrome Depression: CODIACS Vanguard Randomized Controlled Trial ONLINE FIRST • Karina W. Davidson, PhD; J. Thomas Bigger, MD; Matthew M. Burg, PhD; Robert M. Carney, PhD; William F. Chaplin, PhD; Susan Czajkowski, PhD; Ellen Dornelas, PhD; Joan Duer-Hefele, RN; Nancy Frasure-Smith, PhD; Kenneth E. Freedland, PhD; Donald C. Haas, MD, MPH; Allan S. Jaffe, MD; Joseph A. Ladapo, MD, PhD; Francois Lespérance, MD; Vivian Medina, MSW; Jonathan D. Newman, MD, MPH; Gabrielle A. Osorio, BS; Faith Parsons, BS; Joseph E. Schwartz, PhD; Jonathan A. Shaffer, PhD; Peter A. Shapiro, MD; David S. Sheps, MD; Viola Vaccarino, MD, PhD; William Whang, MD, MS; Siqin Ye, MD • JAMA Intern Med. 2013;():1-8. doi:10.1001/jamainternmed.2013.915. doi:10:1001/jama.2010.920
  • 44. Direct-to-consumer television advertising exposure, diagnosis with high cholesterol, and statin use Angina.com: Interview with Dr. Jeff Niederdeppe, Ph.D. Assistant Professor Department of Communication, Cornell University (cont) • Angina.com: What are the main findings of the study? • Dr. Niederdeppe: We examined the relationship between exposure to direct-to-consumer advertising (DTCA) for cholesterol-lowering drugs (most of which are statins) and two clinical variables: diagnosis with high cholesterol, and taking a statin drug in the past year. We found that greater exposure to statin DTCA was associated with an increased likelihood of being diagnosed with high cholesterol and taking a statin to treat it. We also found that these relationships were strongest for adults at comparatively low risk for cardiac events – those who tended to be younger, did not have high blood pressure, and did not smoke regularly. We did not find positive relationships between statin DTCA exposure and high cholesterol diagnosis or statin use for those at higher levels of risk, or those who reported having a history of heart disease or a previous heart attack. • Angina.com: Were any of the findings unexpected? • Dr. Niederdeppe: We were not surprised that statin DTCA exposure would be associated with high cholesterol diagnosis and statin use among lower-risk people, since that pattern has been found before for other types of pharmaceutical drugs. We were surprised, however, at the lack of a positive relationship for those at higher risk. In fact, we even found a negative relationship between DTCA exposure and statin use among high-risk women (those with a history of heart disease, a previous heart attack, and/or diabetes). In other words, greater exposure to statin DTCAs was linked to a lower likelihood of using the drug for this group. These results are puzzling and should be considered preliminary, but we think that future work should see whether or not this pattern exists in other studies with other populations.
  • 45. Direct-to-consumer television advertising exposure, diagnosis with high cholesterol, and statin use Angina.com: Interview with Dr. Jeff Niederdeppe, Ph.D. Assistant Professor Department of Communication, Cornell University • Angina.com: What should clinicians and patients take away from this study? • Dr. Niederdeppe: I’ll first acknowledge that I am not an M.D., and that I understand each patient has a unique set of risk factors, personal and family history, and medical profile. That said, one takeaway point is that we can expect patients to learn about pharmaceutical drugs from DTCA, and that is going to influence the clinical encounter. The average American is exposed to 16 hours of DTCA every year; in contrast they see the doctor an average of 15 minutes per year. Clinicians should be prepared to discuss both the risks and benefits of heavily- advertised pharmaceutical drugs. • I would encourage patients to ask their doctor about the ratio of risks to benefits related to drugs they see advertised. While all DTCAs have to disclose drug risks, the ads tend to do this in a way that focuses greater attention on benefits than risks. • Angina.com: What further research do you recommend as a result of your study? • Dr. Niederdeppe: Many statin DTCAs contain information about lifestyle changes (which are always recommended first to reduce cholesterol) and pharmaceutical drugs. One important question is how these ads might influence people’s lifestyle decisions – whether or not to engage in regular exercise, eat a heart-healthy diet, and maintain a healthy weight. We don’t yet know enough about some of the secondary impacts that these ads may be having on factors that influence the public’s health. • Citation: • Direct-To-Consumer Television Advertising Exposure, Diagnosis with High Cholesterol, and Statin Use. • Niederdeppe J, Byrne S, Avery RJ, Cantor J. • Department of Communication, Cornell University, 328 Kennedy Hall, Ithaca, NY, 14853, USA J Gen Intern Med. 2013 Mar 6. [Epub ahead of print
  • 46. Impact of Socioeconomic Deprivation and Area of Residence on Access to Coronary Revascularization and Mortality After a First Acute Myocardial Infarction in Québec Angina.com Interview with Cardiology Researcher: Stéphane Rinfret MD, SM, FRCPC Institut universitaire de cardiologie et de pneumologie de Québec, Québec City, Québec, Canada • Angina.com: What are the main findings of the study? • Dr. Rinfret : Study shows that patients unfortunately do not persist with dual antiplatelet therapy as we like to think they would do because of the consequences. A simple telephone intervention clearly helped to reach almost perfect adherence. • Angina.com: Were any of the findings unexpected? • Dr. Rinfret : Self-reported adherence is worth nothing. We relied on pharmacy data. When we compared with the answers patients gave to interviewers at 6 months and 1 year, there was very low correlation. Therefore pharmacy data are absolutely required to assess true adherence.
  • 47. Impact of Socioeconomic Deprivation and Area of Residence on Access to Coronary Revascularization and Mortality After a First Acute Myocardial Infarction in Québec Angina.com Interview with Cardiology Researcher: Stéphane Rinfret MD, SM, FRCPC Institut universitaire de cardiologie et de pneumologie de Québec, Québec City, Québec, Canada • Angina.com: What should clinicians and patients take away from this study? • Dr. Rinfret : We need to take responsibility and put systems in place to follow how patients take their medication following stenting. A simple advise not to stop the medication is clearly not enough. • Angina.com: What further research do you recommend as a result of your study? • Dr. Rinfret : As newer DES will likely mandate a shorter DAT duration, a shorter intervention with 1 or 2 calls may be enough. This will have to be tested. • Citation: • Impact of Socioeconomic Deprivation and Area of Residence on Access to Coronary Revascularization and Mortality After a First Acute Myocardial Infarction in Québec • Claudia Blais, PhD, Denis Hamel, MSc, Stéphane Rinfret, MD, SM, FRCPC • Impact of Socioeconomic Deprivation and Area of Residence on Access to Coronary Revascularization and Mortality After a First Acute Myocardial Infarction in Québec • Canadian Journal of Cardiology, Volume 28, Issue 2, March–April 2012, Pages 169–177
  • 48. Comparison of Coronary CT Angiography Image Quality With and Without Breast Shields. Angina.com Interview with Dr. Eddie Hulten, MD MPH Non-Invasive Cardiovascular Imaging Brigham and Women’s Hospital Harvard Medical School • Angina.com: What are the main findings of the study? • Dr. Hulten: We reviewed a convenience sample of patients who had previously been scanned with bismuth breast shields while undergoing coronary CT angiography. We found differences in noise and an association of different plaque types by shielded versus non-shielded groups. Image quality graded on a Likert scale was not significantly different with no difference in uninterpretable segments according to shielded versus not shielded. • Angina.com: Were any of the findings unexpected? • Dr. Hulten: We expected the difference in noise to be more dramatic and the image quality to be poorer for the shielded patients. However, it is important to note that most of the scans were performed using retrospective ECG gating, which delivers a higher overall radiation than most contemporary scans. Thus, the results should not be extrapolated to patients scanned using prospectively-triggered ECG gating or high pitch scans.
  • 49. Comparison of Coronary CT Angiography Image Quality With and Without Breast Shields. Angina.com Interview with Dr. Eddie Hulten, MD MPH Non-Invasive Cardiovascular Imaging Brigham and Women’s Hospital Harvard Medical School (cont) • Angina.com: What should clinicians and patients take away from this study? • Dr. Hulten: Patients scanned with breast shields in this study had interpretable coronary CT angiograms although the scans were noisier and had different plaque types detected, which could be associated with the shielding. However, breast shields are not currently recommended for clinical use by the SCCT or medical physicists due to concern for wasting radiation. Also other radiation sparing techniques may be more effective, reproducible, and impact image noise less. • Angina.com: What further research do you recommend as a result of your study? • Dr. Hulten: Study of additional radiation reducing methods during coronary CT angiography is indicated. While breast shields are not recommended for routine clinical use, further evaluation of shields’ impact upon coronary CT imaging may be warranted in specific research protocols. • Citation: • Comparison of Coronary CT Angiography Image Quality With and Without Breast Shields. • Hulten E, Devine P, Welch T, Feuerstein I, Taylor A, Petrillo S, Luncheon M, Nguyen B, Villines TC. • Cardiology Service, Walter Reed National Military Medical Center, Bethesda, MD. AJR Am J Roentgenol. 2013 Mar;200(3):529-36. doi: 10.2214/AJR.11.8302.
  • 50. Trajectories of cardiometabolic risk factors before diagnosis of three subtypes of type 2 diabetes Angina.com Interview with: Dr Kristine Færch PhD BSc, MSc (Human Nutrison), PhD. STAR Researcher Steno Diabetes Center Niels Steensens Vej 1 DK-2820 Gentofte Denmark • Angina.com: What are the main findings of the study? • Dr. Færch: Our study underscores the heterogeneity of type 2 diabetes. Individuals who develop type 2 diabetes have different underlying mechanisms dependent on whether they are diagnosed with diabetes by elevated fasting glucose, elevated 2-hour glucose (after an oral glucose tolerance test) or combined elevated fasting and 2-hour glucose. We found very different trajectories of beta cell function, obesity development and several cardiovascular risk factors prior to diagnosis of diabetes in the three groups.
  • 51. Trajectories of cardiometabolic risk factors before diagnosis of three subtypes of type 2 diabetes Angina.com Interview with: Dr Kristine Færch PhD BSc, MSc (Human Nutrison), PhD. STAR Researcher Steno Diabetes Center Niels Steensens Vej 1 DK-2820 Gentofte Denmark (cont) • Angina.com: Were any of the findings unexpected? • Dr. Færch: We expected to find different patterns of some cardiometabolic risk factors, but we were surprised that the differences were so pronounced in the different subtypes of type 2 diabetes – especially with regard to beta cell function. A classic pattern of early beta cell compensation followed by a progressive loss of beta cell function was only found in 25% of the individuals who developed diabetes, whereas 20% had a constant low beta cell function starting already 18 years before they were diagnosed with diabetes. • Angina.com: What should clinicians and patients take away from this study? • Dr. Færch: Most clinicians already acknowledge that type 2 diabetes is a heterogeneous disease, but this study adds to a deeper understanding of this heterogeneity. It is likely that the optimal treatment will differ among patients with the different subtypes identified in our study.
  • 52. Trajectories of cardiometabolic risk factors before diagnosis of three subtypes of type 2 diabetes Angina.com Interview with: Dr Kristine Færch PhD BSc, MSc (Human Nutrison), PhD. STAR Researcher Steno Diabetes Center Niels Steensens Vej 1 DK-2820 Gentofte Denmark (cont) • Angina.com: What further research do you recommend as a result of your study? • Dr. Færch: First of all, it is important to study the underlying mechanisms of the different type 2 diabetes subtypes in other populations because there may be ethnic differences. If our findings are confirmed in other populations, the next step will be to see how the different subtypes of type 2 diabetes respond to medication in regard to glucose control and prevention of diabetic complications. Also, it will be relevant to examine whether type 2 diabetes should be prevented differently in high-risk individuals with impaired fasting glycemia (elevated fasting glucose) and impaired glucose tolerance (elevated 2-hour glucose). • Citation: • Trajectories of cardiometabolic risk factors before diagnosis of three subtypes of type 2 diabetes: a post-hoc analysis of the longitudinal Whitehall II cohort study Dr Kristine Færch PhD,Daniel R Witte PhD,Adam G Tabák PhD,Leigh Perreault MD,Christian Herder PhD,Eric J Brunner PhD,Prof Mika Kivimäki PhD,Dorte Vistisen PhD The Lancet Diabetes & Endocrinology – 21 February 2013 DOI: 10.1016/S2213-8587(13)70008-1
  • 53. African American Race is a Correlate of Heart Failure in Breast Cancer Survivors: A study of 26,347 women identified with breast cancer from 1973-2007 Angina.com Interview with Anna Valina-Toth, MD, PhD Department of Internal Medicine Case Western Reserve University at MetroHealth • Angina.com: What are the main findings of the study? • Answer: – In female breast cancer survivors, the risk of developing heart failure is higher in African Americans compared to Caucasians. • Angina.com: Were any of the findings unexpected? • Answer: - We are not surprised that in breast cancer survivors, African Americans have higher heart failure occurrence compared to whites. African Americans, especially women, relative to whites, have excessive prevalence of hypertension, obesity and obesity related diseases (including diabetes), oxidative stress, inflammation, and endothelium nitric oxide-dependent vascular dysfunction and vitamin D deficiency. • All these aforementioned co-morbidities plausibly predispose African Americans to higher risk of HF events including those who are breast cancer survivors.
  • 54. African American Race is a Correlate of Heart Failure in Breast Cancer Survivors: A study of 26,347 women identified with breast cancer from 1973-2007 Angina.com Interview with Anna Valina-Toth, MD, PhD Department of Internal Medicine Case Western Reserve University at MetroHealth (cont) • Angina.com: What should clinicians and patients take away from this study? • Answer:- This study reveals a significant health disparity in heart failure prevalence that disproportionately affects African Americans compared to Caucasians in a cohort of breast cancer survivors. • Angina.com: What further research do you recommend as a result of your study? • Answer:- Future research to determine whether non-invasive cardiac imaging and pre-treatment with cardioprotective drugs such as ACE-inhibitors and beta blockers prior to initiation of antracyclines and trastuzumab based-chemotherapy would reduce the risk of heart failure in breast cancer patients especially in African Americans who are predisposed to developing heart failure merit investigation. • Citation: • African American Race is a Correlate of Heart Failure in Breast Cancer Survivors: A study of 26,347 women identified with breast cancer from 1973-2007 Presented at ACC 2013 March 2013
  • 55. Tooth Loss Is Highly Prevalent And Associated With Cardiovascular Risk Factors In Patients With Chronic Coronary Heart Disease In The Global Stability Trial Angina.com Interview with Dr. Ola Vedin, M.D. Department of Medical Sciences/Uppsala Clinical Research Centre (UCR) Uppsala University Uppsala SWEDEN • Angina.com: What are the main findings of the study? • Dr. Vedin: Self-reported tooth loss and gum bleeding, as markers of periodontal disease, are common in this global population of patients with established coronary heart disease (i.e. previous myocardial infarction or verified coronary artery stenosis). Poor dental health, especially tooth loss, is associated with several established cardiovascular risk factors, including diabetes, smoking, high blood pressure and obesity, but also with more novel ones including Lp-PLA2. Angina.com: Were any of the findings unexpected?
 • Dr. Vedin: We were surprised by the large proportion of patients with no or very few teeth and had expected somewhat stronger associations between gum bleeding and cardiovascular risk factors.
  • 56. Tooth Loss Is Highly Prevalent And Associated With Cardiovascular Risk Factors In Patients With Chronic Coronary Heart Disease In The Global Stability Trial Angina.com Interview with Dr. Ola Vedin, M.D. Department of Medical Sciences/Uppsala Clinical Research Centre (UCR) Uppsala University Uppsala SWEDEN (cont) • Angina.com: What should clinicians and patients take away from this study? • Dr. Vedin: There is an association between self-reported tooth loss and cardiovascular risk factors but we do not yet know whether this relationship is causal. • Periodontal disease in relation to cardiovascular risk is a hot, but poorly understood topic that needs further exploring.
  • 57. Tooth Loss Is Highly Prevalent And Associated With Cardiovascular Risk Factors In Patients With Chronic Coronary Heart Disease In The Global Stability Trial Angina.com Interview with Dr. Ola Vedin, M.D. Department of Medical Sciences/Uppsala Clinical Research Centre (UCR) Uppsala University Uppsala SWEDEN (cont) • Angina.com: What further research do you recommend as a result of your study? • Dr. Vedin: We are anticipating outcome data later this year when the study closes. Then we can relate the data on dental health to cardiovascular outcome and, after adjusting for common risk factors, clarify whether self-reported tooth loss and gum bleeding may actually be associated with myocardial infarction and death. • More widely speaking, there is a need for independent studies designed to determine whether poor dental health, or periodontal disease, actually causes coronary heart disease and if so, if periodontal treatment lowers cardiovascular risk. • Citation: • Tooth Loss Is Highly Prevalent And Associated With Cardiovascular Risk Factors In Patients With Chronic Coronary Heart Disease In The Global Stability Trial • Dr. Vedin presented the study “Tooth Loss Is Highly Prevalent And Associated With Cardiovascular Risk Factors In Patients With Chronic Coronary Heart Disease In The Global Stability Trial” on Saturday, March 9 at 3:45 p.m., in Moscone Center, Expo North during the ACC13 meeting
  • 58. Sleep Apnea and Risk of Peptic Ulcer Bleeding Angina.com Interview with: Chou, Kun-Ta, M.D Taipei Veterans General Hospital, Taipei, Taiwan • Angina.com: What are the main findings of the study? • Response: In this retrospective matched-control cohort study including 35,480 patients (7,096 sleep apnea patients vs. 28,384 controls) with a mean follow-up period of 3.57 years, we found patients with sleep apnea experienced a 2.400- fold (95% CI, 1.544-3.731, p<0.001) higher risk for incident peptic ulcer bleeding after adjusting other significant variables.
  • 59. Sleep Apnea and Risk of Peptic Ulcer Bleeding Angina.com Interview with: Chou, Kun-Ta, M.D Taipei Veterans General Hospital, Taipei, Taiwan (cont) • Angina.com: Were any of the findings unexpected? • Response: Sleep apnea is a common disease and linked with a variety of cardiovascular diseases, such as hypertension, atherosclerosis, and arrhythmia. However, its impact on the affected individuals seems more widespread than we would expect, not exclusively limited to the cardiovascular system. The gastrointestinal system is probable to be affected as well. We found it surprising that sleep apnea confers a higher risk for incident peptic ulcer bleeding, even within a relatively short follow-up period in such a young population (aged 46.61 years on average).
  • 60. Sleep Apnea and Risk of Peptic Ulcer Bleeding Angina.com Interview with: Chou, Kun-Ta, M.D Taipei Veterans General Hospital, Taipei, Taiwan (cont) • Angina.com: What should clinicians and patients take away from this study? • Response: Our results suggest that sleep apnea is an independent risk factor for peptic ulcer bleeding. Despite clear conclusion provided, this study is limited by its retrospective design and inability to include information about the lifestyle of the enrollees, such as smoking, alcohol consumption, obesity etc., which may possibly confound our results. • For treatment of peptic ulcer bleeding in clinical practice, the clinicians should go through the traditional risk factors in such patients. For those without apparent risk factors identified, we may survey for sleep apnea as a potential predisposing factor.
  • 61. Sleep Apnea and Risk of Peptic Ulcer Bleeding Angina.com Interview with: Chou, Kun-Ta, M.D Taipei Veterans General Hospital, Taipei, Taiwan (cont) • Response: What further research do you recommend as a result of your study? • Response: Our study may prompt further elaborate prospective research to confirm our conclusion and probe into the mechanism underlying the linkage of sleep apnea and its gastrointestinal complications. • Citation: • Sleep Apnea and Risk of Peptic Ulcer Bleeding: A Nationwide Population-based Study • Am J Med. 2013 Mar;126(3):249-255.e1. doi: 10.1016/j.amjmed.2012.08.017. • Shiao TH, Liu CJ, Luo JC, Su KC, Chen YM, Chen TJ, Chou KT, Shiao GM, Lee YC. Department of Chest Medicine, Taipei Veterans General Hospital, Taipei, Taiwan.
  • 62. The Effect of Weight Loss on the Incidence of Heart Failure or Death in MADIT-CRT Patients Angina.com Interview with Dr. Valentina Kutyifa, MD, MSc University of Rochester Medical Center NY • Angina.com: What are the main findings of the study? • Dr. Kutyifa: In 170 of 994 (17%) patients implanted with CRT-D in the MADIT-CRT clinical trial, an unplanned, greater than – 2 kg weight loss was observed at 12 months after device implantation. The risk of heart failure or death was significantly higher in patients with unanticipated, significant weight loss as compared to those without weight loss. Each kilogram weight loss was associated with 4% increase in the risk of heart failure or death during an average of 29 months follow-up in the study. Patients with left bundle branch block and significant weight loss (> – 2kg) doubled their risk of heart failure or death.
  • 63. The Effect of Weight Loss on the Incidence of Heart Failure or Death in MADIT-CRT Patients Angina.com Interview with Dr. Valentina Kutyifa, MD, MSc University of Rochester Medical Center NY (cont) • Angina.com: Were any of the findings unexpected? • Dr. Kutyifa: This study is consistent with the findings from other studies that have looked at the obesity paradox, suggesting that patients with weight loss have worse clinical outcome as compared to those without a weight loss. We need to stress that in this particular study, patients were not following any dietary rules, and they were not encouraged to lose weight, the weight loss was unanticipated. • Angina.com: What should clinicians and patients take away from this study? • Dr. Kutyifa: In patients with heart failure who receive a cardiac resynchronization device with defibrillator (CRT-D), unintended weight loss increases the risk of heart failure or death. • It may be advisable for heart failure patients with unplanned significant weight loss to consult their doctor, and physicians may perform diagnostic tests, refining medical treatment if necessary, and monitor these patients more closely to prevent the development of serious events. • However, we still suggest implementing weight control under a physician’s supervision in obese heart failure patients, because obesity is known to be associated with diabetes, high blood pressure, and the development of coronary artery disease.
  • 64. The Effect of Weight Loss on the Incidence of Heart Failure or Death in MADIT-CRT Patients Angina.com Interview with Dr. Valentina Kutyifa, MD, MSc University of Rochester Medical Center NY (cont) • Angina.com: What further research do you recommend as a result of your study? • Dr. Kutyifa: The effects of weight loss needs to be further evaluated in patients with an implanted CRT-D, and baseline obesity needs to be taken into consideration. It would be interesting to see, if this effect is maintained regardless of the baseline weight, and baseline body mass index (BMI), or less obese patients would more affected than those with obesity to start with. • Citation: • The Effect of Weight Loss on the Incidence of Heart Failure or Death in MADIT-CRT Patients • American College of Cardiology annual meeting in San Francisco Abstract: 308 Valentina Kutyifa ,Mehmet Aktas, David Huang, Scott McNitt, Arthur Moss, Wojciech Zareba, University of Rochester Medical Center, Heart Research FollowUp Program, Rochester, NY, USA
  • 65. Association of Body Mass Index with Risks of Myocardial Infarction and Ischemic Stroke in Young Women Angina.com Interview with Michelle Schmiegelow PhD student at the University of Copenhagen • Angina.com: What are the main findings of the study? • Answer: In this nationwide register-based study of 273,000 healthy women with a median age of approximately 30 years, we found obesity (body mass index >30 kg/m2) to be associated with a significantly increased risk of myocardial infarction and ischemic stroke within 4.5 years following childbirth. The absolute risks were low, but the key message is clear. • Angina.com: Were any of the findings unexpected? • Answer: We found it surprising that the obesity-associated health risk becomes apparent within a relatively short follow-up in such a young population of women even following adjustment for important cardiovascular risk factors.